GI/GU Exam 3

Ace your homework & exams now with Quizwiz!

Complications of prostate surgery

#1 hemorrhage infection bladder spasms urinary retention erectile dysfunction

Interventions for hemorrhoids

1. Apply cold packs to the anal-rectal area followed by sitz baths as prescribed. 2. Apply witch hazel soaks and topical anesthetics as prescribed. 3. Encourage a high-fiber diet and fluids to promote bowel movements without straining. 4. Administer stool softeners as prescribed.

What is the primary nursing diagnosis for a client with a bowel obstruction? 1. Deficient fluid volume 2. Deficient knowledge 3. Acute pain 4. Ineffective tissue perfusion

1. Deficient fluid volume Feces, fluid, and gas accumulate above a bowel obstruction. Then the absorption of fluids decreases and gastric secretions increase. This process leads to a loss of fluids and electrolytes in circulation. Options 2, 3, and 4 are applicable but not the primary nursing diagnosis.

The nurse is reviewing the record of a client with Crohn's disease. Which stool characteristic should the nurse expect to note documented in the client's record? 1. Diarrhea 2. Chronic constipation 3. Constipation alternating with diarrhea 4. Stool constantly oozing from the rectum

1. Diarrhea Crohn's disease is characterized by nonbloody diarrhea of usually not more than four to five stools daily. Over time, the diarrhea episodes increase in frequency, duration, and severity. Options 2, 3, and 4 are not characteristics of Crohn's disease.

Risk factors for nephrolithiasis

1. Family history of stone formation 2. Diet high in calcium, vitamin D, protein, oxalate, purines, or alkali 3. Obstruction and urinary stasis 4. Dehydration 5. Use of diuretics, which can cause volume depletion 6. Urinary tract infections and prolonged urinary catheterization 7. Immobilization 8. Hypercalcemia and hyperparathyroidism 9. Elevated uric acid level, such as in gout

Which infection control equipment is necessary for the client diagnosed with Clostridium difficile diarrhea? 1. Gloves 2. Mask 3. Face shield 4. N-95 respirator

1. Gloves The client diagnosed with Clostridium difficile diarrhea requires contact isolation. Contact isolation precautions require the use of glove and a gown if soiling is likely. A mask, face shield, and N-95 respirator aren't necessary to maintain contact isolation.

Following perineal surgery, a client is at risk for a wound infection related to incontinence. The correct management of this problem is to: 1. Insert a continuous indwelling catheter per order. 2. Assist to the toilet and protect the skin with cream. 3. Limit oral fluid intake. 4. Give a loop diuretic, such as furosemide, as ordered.

1. Insert a continuous indwelling catheter per order. An indwelling catheter diverts urine away from the operative site, reducing risk of wound infection.

Functions of the kidney

1. Maintain acid-base balance 2. Excrete end products of body metabolism 3. Control fluid and electrolyte balance 4. Excrete bacterial toxins, water-soluble drugs, and drug metabolites 5. Secrete renin to regulate the blood pressure and erythropoietin to stimulate the bone marrow to produce red blood cells. 6. Synthesize vitamin D for calcium absorption and regulation of the parathyroid hormones.

Assessment findings for polycystic kidney disease

1. Often asymptomatic until the ages of 30 to 40 years 2. Flank, lumbar , or abdominal pain that worsens with activity and is relieved when lying 3. Fever and chills 4. Recurrent urinary tract infections 5. Hematuria, proteinuria, pyuria 6. Calculi 7. Hypertension 8. Palpable abdominal masses and enlarged kidneys

Assessment findings for appendicitis

1. Pain in the periumbilical area that descends to the right lower quadrant 2. Abdominal pain that is most intense at McBurney's point 3. Rebound tenderness and abdominal rigidity 4. Low-grade fever 5. Elevated white blood cell count 6. Anorexia, nausea, and vomiting 7. Client in side-lying position, with abdominal guarding and legs flexed 8. Constipation or diarrhea

The nurse would position a client with ruptured appendix in: 1. Semi-Fowler's. 2. Trendelenburg. 3. Left Sims'. 4. Dorsal recumbent.

1. Semi-Fowler's. The client is placed in a semi-Fowler's position to promote the flow of drainage to the pelvic region, where a localized abscess can be frained or resolved by the body's normal defenses. The elevated position also keeps the infection from spreading upward in the peritoneal cavity.

The nurse is planning to administer a sodium polystyrene sulfonate (Kayexalate) enema to a client with a potassium level of 6.2 mEq/L. Correct administration and the effects of this enema would include having the client: 1. retain the enema for 30 minutes to allow for sodium exchange; afterward, the client should have diarrhea. 2. retain the enema for 30 minutes to allow for glucose exchange; afterward, the client should have diarrhea. 3. retain the enema for 60 minutes to allow for sodium exchange; diarrhea isn't necessary to reduce the potassium level. 4. retain the enema for 60 minutes to allow for glucose exchange; diarrhea isn't necessary to reduce the potassium level.

1. retain the enema for 30 minutes to allow for sodium exchange; afterward, the client should have diarrhea. Kayexalate is a sodium-exchange resin. Thus, the client will gain sodium as potassium is lost in the bowel. For the exchange to occur, Kayexalate must be in contact with the bowel for at least 30 minutes. Sorbitol in the Kayexalate enema causes diarrhea, which increases potassium loss and decreases the potential for Kayexalate retention.

Which laboratory test is the most accurate indicator of a client's renal function? 1. Blood urea nitrogen 2. Creatinine clearance 3. Serum creatinine 4. Urinalysis

2. Creatinine clearance Creatinine clearance is the most accurate indicator of a client's renal function because it closely correlates with the kidney's glomerular filtration rate and tubular excretion ability. Results from the other options may be influenced by various conditions and aren't specific to renal disease.

Following an ileostomy, when should the drainage appliance be applied to the stoma? 1. 24 hours later, when edema has subsided. 2. In the operating room. 3. After the ileostomy begins to function. 4. When the client is able to begin self-care procedures.

2. In the operating room. Drainage from the ileostomy contains secretions that are rich in digestive enzymes and highly irritating to the skin. Protection of the skin from the effects of these enzymes is begun at once. Skin exposed to these enzymes even for a short time becomes reddened, painful, and excoriated.

The nurse explains to the client that a flecible sigmoidoscopy involves: 1. Instillation of a radiopaque contrast medium into the lower GI tract. 2. Insertion of a fiberoptic scope that allows for direct visual examination of the anal canal, rectum, and sigmoid colon. 3. Insertion of a fiberoptic scope that allows for direct visualization of the sigmoid colon, transverse colon and ileocecal valve. 4. Surgical removal of polyps and biopsy of suspicious GI mucosa.

2. Insertion of a fiberoptic scope that allows for direct visual examination of the anal canal, rectum, and sigmoid colon.

The mouth care measure that should be used with caution by the nurse when a client has a nasogastric tube is: 1. Regularly brushing teeth and tongue with soft brush. 2. Sucking on ice chips to relieve dryness. 3. Occasionally rinsing mouth with a nonastringent substance and massaging gums. 4. Application of lemon juice and glycerine swabs to the lips.

2. Sucking on ice chips to relieve dryness. The client should be cautioned to limit the number of ice chips he or she sucks on. The NG tube will remove not only the increased water ingested from the melted chips but also essential electrolytes.

The nurse explains to a client that the colostomy appliance for a double-barrel colostomy should be changed: 1. Every day. 2. When drainage leaks through the seal. 3. Once a week. 4. At a time selected by the visiting nurse.

2. When drainage leaks through the seal. Every day is too often and can damage the skin from pulling the appliance off so often. The appliance should be changes every 2-3 days or as soon as there is leakage. Drainage can excoriate the skin; therefore a new appliance needs to be applied as soon as leakage appears. 1 week is too long for the skin to go without being examined and cleansed.

The nurse is performing an assessment on a client who has developed a paralytic ileus. The client's bowel sounds will be: 1. hyperactive. 2. hypoactive. 3. high-pitched. 4. blowing.

2. hypoactive. If a paralytic ileus occurs, bowel sounds will be hypoactive or absent. Hyperactive bowel sounds may signify hunger, intestinal obstruction, or diarrhea. High-pitched sounds may signify a dilated bowel. A blowing sound may be a bruit from a partially obstructed abdominal aorta.

A client with an indwelling urinary catheter is suspected of having a urinary tract infection. The nurse should collect a urine specimen for culture and sensitivity by: 1. disconnecting the tubing from the urinary catheter and letting the urine flow into a sterile container. 2. wiping the self-sealing aspiration port with antiseptic solution and aspirating urine with a sterile needle. 3. draining urine from the drainage bag into a sterile container. 4. clamping the tubing for 60 minutes and inserting a sterile needle into the tubing above the clamp to aspirate urine.

2. wiping the self-sealing aspiration port with antiseptic solution and aspirating urine with a sterile needle. Most catheters have a self-sealing port for obtaining a urine specimen. Antiseptic solution is used to reduce the risk of introducing microorganisms into the catheter. Tubing shouldn't be disconnected from the urinary catheter. Any break in the closed urine drainage system may allow the entry of microorganisms. Urine in urine drainage bags may not be fresh and may contain bacteria, giving false test results. When there is no urine in the tubing, the catheter may be clamped for no more than 30 minutes to allow urine to collect.

Paralytic ileus is described by the nurse as: 1. Edema of the intestinal mucosa. 2. Acute dilation of the colon. 3. Absent, diminished, or uncoordinated autonomic stimulation of peristalsis. 4. High-pitched, tinkling bowel sounds over the area of obstruction.

3. Absent, diminished, or uncoordinated autonomic stimulation of peristalsis.

A client has a colostomy following surgery for colon cancer. Which assessment finding would require immediate action by a nurse? 1. Stoma raised 2 cm from the abdominal wall. 2. Bleeding was noted from the stoma during care. 3. Complaint of pain with light touch. 4. Stoma was pinkish-red in color.

3. Complaint of pain with light touch. The stoma will be 1 to 2 cm above the skin. Some bleeding is normal. There are no nerves in the mucous membranes, there should be no pain when touching. Pain would possibly indicate a problem internally.

The nurse should know that the drainage is normal 4 days after a sigmoid colostomy when the material is: 1. Green liquid. 2. Solid formed. 3. Loose, bloody. 4. Semiformed.

3. Loose, bloody. Normal bowel function and soft-formed stool usually do not occur until around the seventh day following surgery.

The nurse is irrigating a colostomy when the client complains of cramping. What is the most appropriate initial action for the nurse? 1. Increase the flow of solution. 2. Ask the client to turn to his other side. 3. Pinch the tubing the interrupt the flow of solution. 4. Remove the tube from the colostomy.

3. Pinch the tubing the interrupt the flow of solution.

Nursing assessment of a client with peritonitis (acute or chronic inflammation of the peritoneum) reveals hypotension, tachycardia, and signs and symptoms of dehydration. The nurse also expects to find: 1. tenderness and pain in the right upper abdominal quadrant. 2. jaundice and vomiting. 3. severe abdominal pain with direct palpation or rebound tenderness. 4. rectal bleeding and a change in bowel habits.

3. severe abdominal pain with direct palpation or rebound tenderness. Peritonitis decreases intestinal motility and causes intestinal distention. A classic sign of peritonitis is a sudden, diffuse, severe abdominal pain that intensifies in the area of the underlying causative disorder (such as appendicitis, diverticulitis, ulcerative colitis, or a strangulated obstruction). The client also has direct or rebound tenderness. Tenderness and pain in the right upper abdominal quadrant suggest cholecystitis. Jaundice and vomiting are signs of cirrhosis of the liver. Rectal bleeding or a change in bowel habits may indicate colorectal cancer.

A client has a newly created colostomy. After participating in counseling with the nurse and receiving support from the spouse, the client decides to change the colostomy pouch unaided. Which behavior suggests that the client is beginning to accept the change in body image? 1. The client closes the eyes when the abdomen is exposed. 2. The client avoids talking about the recent surgery. 3. The client asks the spouse to leave the room. 4. The client touches the altered body part.

4. The client touches the altered body part. By touching the altered body part, the client recognizes the body change and establishes that the change is real. Closing the eyes, not looking at the abdomen when the colostomy is exposed, or avoiding talking about the surgery reflects denial, instead of acceptance of the change. Asking the spouse to leave the room signifies that the client is ashamed of the change and not coping with it.

Crohn's disease

A chronic progressive disease of the liver characterized by diffuse degeneration and destruction of hepatocytes. Repeated destruction of hepatic cells causes the formation of scar tissue.

Kock pouch

A continent internal ileal reservoir created from a segment of the ileum and ascending colon. The ureters are implanted into the side of the reservoir, and a special nipple valve is constructed to attach the reservoir to the skin. Postoperatively, the client will have a Foley catheter in place to drain urine continuously until the pouch has healed. The catheter is irrigated gently with normal saline to prevent obstruction from mucus or clots. Following removal of the catheter, the client is instructed in how to self-catheterize and to drain the reservoir at 4- to 6-hour intervals

Polycystic kidney disease

A cystic formation and hypertrophy of the kidneys, which leads to cystic rupture, infection, formation of scar tissue, and damaged nephrons There is no specific treatment to arrest the progress of the destructive cysts. The ultimate result of this disease is renal failure.

Extracorporeal shock wave lithotripsy (ESWL)

A noninvasive mechanical procedure for breaking up stones located in the kidney or upper ureter so that they can pass spontaneously or be removed by other methods No incision is made and no drains are placed; a stent may be placed to facilitate passing stone fragments. Fluoroscopy is used to visualize the stone and ultrasonic waves are delivered to the area of the stone to disintegrate it. The stones are passed in the urine within a few days.

Continuous Bladder Irrigation

A three-way (lumen) irrigation is used to decrease bleeding and to keep the bladder free from clots—one lumen is for inflating the balloon (30 mL); one lumen is for instillation (inflow); one lumen is for outflow.

Pyelonephritis

An inflammation of the renal pelvis and the parenchyma commonly caused by bacterial invasion. Acute pyelonephritis often occurs after bacterial contamination of the urethra or following an invasive procedure of the urinary tract. Chronic pyelonephritis most commonly occurs following chronic urinary flow obstruction with reflux. Escherichia coli is the most common causative bacterial organism.

KUB (kidneys, ureters, and bladder) radiography

An x-ray of the urinary system and adjacent structures to detect urinary calculi.

Diagnostics for peritonitis

CBC serum electrolytes x-ray, CT, ultrasound paracentesis

Questions to ask a patient with diarrhea

History of travel? Diet? Medications? Previous surgery? Family history? Personal contacts?

Functional incontinence

Lower urinary tract function is intact but other factors, such as severe cognitive impairment (eg, Alzheimer's dementia), make it difficult for the patient to identify the need to void or physical impairments make it difficult or impossible for the patient to reach the toilet in time for voiding.

Interventions for intestinal obstruction

NPO NG tube (irrigate with normal saline) Intestinal tube enema IV fluids (NS with K+) TPN surgery (preop neomycin enema)

Transurethral resection of the prostate (TURP)

Removal of benign prostatic tissue surrounding the urethra with use of a resectoscope introduced through the urethra; there is little risk of impotence and it is most commonly used for BPH.

strangulated hernia

irreducible with obstruction to blood supply - surgical emergency

Nephrostomy tube

temporary - used to preserve renal function inserted into pelvis of kidney

Surgical interventions for ulcerative colitis

Total proctocolectomy with permanent ileostomy Kock ileostomy (continent ileostomy) Ileoanal reservoir

Complications of BPH

UTIs Hydronephrosis Pyelonephritis

ulcerative colitis

Ulcerative and inflammatory disease of the bowel that results in poor absorption of nutrients. Acute ulcerative colitis results in vascular congestion, hemorrhage, edema, and ulceration of the bowel mucosa. Chronic ulcerative colitis causes muscular hypertrophy, fat deposits, and fibrous tissue with bowel thickening, shortening, and narrowing.

Benign prostatic hypertrophy (benign prostatic hyperplasia; BPH)

a slow enlargement of the prostate gland, with hypertrophy and hyperplasia of normal tissue. Enlargement compresses the urethra, resulting in partial or complete obstruction. Usually occurs in men older than 50 years.

PostOp ileostomy

a. Healthy stoma is red; a color change to dark blue or black should be reported to the physician. b. Postoperative drainage will be dark green and progress to yellow as the client begins to eat. c. Stool is liquid. d. Risk for dehydration and electrolyte imbalance exists. e. Do not administer medications such as suppositories through an ileostomy.

Hernia

abnormal protrusion of an organ through the structures normally containing it

Diagnostic tests for diarrhea

blood (CBC, LFTs, iron/folate) stool culture (WBCs, blood, ova & parasites) endoscopy GI series w/ barium swallow

Causes of peritonitis

blood born infection cirrhosis of the liver perforation/rupture of bowel/appendix pancreatitis peritoneal dialysis abdominal surgery

reducible hernia

can be manually replaced into cavity

Causes for cystectomy with urinary diversion

cancer of bladder neurogenic bladder congenital anomalies trauma chronic infection

Causes of diarrhea

decreased fluid absorption - laxative abuse, mucosal damage (Chron's, radiation, colitis, ischemic bowel dis.) increased fluid secretion - infectious bacteria endotoxins, antibiotics, foods w/ sorbitol, hormonal, adenoma of pancreas motility disturbances - IBS, gastrectomy

Complications of diarrhea

dehydration electrolyte imbalance

Mixed urinary incontinence

encompasses several types of urinary incontinence, is involuntary leakage associated with urgency and also with exertion, effort, sneezing, or coughing

Diet for fecal incontinence

high fiber increased fluid

Complications of peritonitis

hypovolemic shock septicemia paralytic ileus organ failure

Complications of bowel obstruction

hypovolemic shock - due to necrosis and rupture of bowel - fluid shifts into peritoneum fluid/electrolyte and acid/base imbalance - metabolic alkalosis high in bowel - metabolic acidosis lower in bowel dehydration - from vomiting strangulation and gangrene of bowel

Interventions for constipation

laxatives (use cautiously) enemas (oil retention - softens feces, hypotonic, phosphate) high fiber food increased fluids (at least 3,000mL/day)

Causes of fecal incontinence

motor - muscle contraction sensory - dementia, stroke, spinal cord injury, degenerative disease

Assessment findings for intestinal obstruction

nausea abdominal distention obstipation high pitched bowel sounds temp > 100 SBO projectile vomiting pain relieved with vomiting LBO persistent colicky pain no vomiting orange brown stool

Treatment for polycystic kidney disease

nephrectomy dialysis

Complications of Chron's disease

scar tissue with narrowing & SBO fistulas between bowel and urinary tract bowel perforation/peritonitis fat malabsorption deficiencies (A,D,E,K) gluten intolerance cholelitiasis & nephrolithiasis (due to oxalate absorption)

Nephrosclerosis

sclerosis of the small arteries and arterioles of the kidney usually caused by vascular changes due to malignant hypertension and arteriosclerosis treated with antihypertensives

Iatrogenic incontinence

the involuntary loss of urine due to extrinsic medical factors, predominantly medications

Interventions for hernias

truss scrotal support avoid coughing no heavy lifting surgery

Complications of constipation

valsalva maneuver & syncope divertulosis obstipation bowel perforation

One hour before a client is to undergo abdominal surgery, the physician orders atropine, 0.6 mg I.M. The client asks the nurse why this drug must be administered. How should the nurse respond? 1. "Atropine decreases salivation and gastric secretions." 2. "Atropine controls the heart rate and blood pressure." 3. "Atropine improves ventilation by increasing the respiratory rate." 4. "Atropine enhances the effect of anesthetic agents."

1. "Atropine decreases salivation and gastric secretions." When used as preanesthesia medications, atropine and other cholinergic blocking agents reduce salivation and gastric secretions, thus helping to prevent aspiration of secretions during surgery. Atropine increases the heart rate and cardiac contractility, decreases bronchial secretions, and causes bronchodilation. No evidence indicates that the drug enhances the effect of anesthetic agents.

A client is scheduled for a renal clearance test. The nurse should explain that this test is done to assess the kidneys' ability to remove a substance from the plasma in: 1. 1 minute. 2. 30 minutes. 3. 1 hour. 4. 24 hours.

1. 1 minute. The renal clearance test determines the kidneys' ability to remove a substance from the plasma in 1 minute. It doesn't measure the kidneys' ability to remove a substance over a longer period.

A client comes to the emergency department complaining of severe pain in the right flank, nausea, and vomiting. The physician tentatively diagnoses right ureterolithiasis (renal calculi). When planning this client's care, the nurse should assign highest priority to which nursing diagnosis? 1. Acute pain 2. Risk for infection 3. Impaired urinary elimination 4. Imbalanced nutrition: Less than body requirements

1. Acute pain Ureterolithiasis typically causes such acute, severe pain that the client can't rest and becomes increasingly anxious. Therefore, the nursing diagnosis of Acute pain takes highest priority. Diagnoses of Risk for infection and Impaired urinary elimination are appropriate when the client's pain is controlled. A diagnosis of Imbalanced nutrition: Less than body requirements isn't pertinent at this time.

Assessment findings for ulcerative colitis

1. Anorexia 2. Weight loss 3. Malaise 4. Abdominal tenderness and cramping 5. Severe diarrhea that may contain blood and mucus 6. Malnutrition, dehydration, and electrolyte imbalances 7. Anemia 8. Vitamin K deficiency

Assessment findings for diverticulitis

1. Left lower quadrant abdominal pain that increases with coughing, straining, or lifting 2. Elevated temperature 3. Nausea and vomiting 4. Flatulence 5. Cramp-like pain 6. Abdominal distention and tenderness 7. Palpable, tender rectal mass may be present. 8. Blood in the stools

A client with ulcerative colitis is receiving methaneline bromide (Banthine). This nurse knows the primary reason this client is receiving methaneline bromide is to: 1. Supress inflammation of the bowel. 2. Reduce peristaltic activity. 3. Neutralize acid in the gastrointestinal tract. 4, Increase bowel tone.

2. Reduce peristaltic activity.

The nurse is teaching a client with malabsorption syndrome about the disorder and its treatment. The client asks which part of the GI tract absorbs food. The nurse tells the client that products of digestion are absorbed mainly in the: 1. stomach. 2. small intestine. 3. large intestine. 4. rectum.

2. small intestine. The small intestine absorbs products of digestion, completes food digestion, and secretes hormones that help control the secretion of bile, pancreatic juice, and intestinal secretions. The stomach stores, mixes, and liquefies the food bolus into chyme and controls food passage into the duodenum; it doesn't absorb products of digestion. Although the large intestine completes the absorption of water, chloride, and sodium, it plays no part in absorbing food. The rectum is the portion of the large intestine that forms and expels feces from the body; its functions don't include absorption.

A client who has been treated for diverticulitis is being discharged on oral propantheline bromide (Pro-Banthine). The nurse should instruct the client to take the drug at which times? 1. With meals and at bedtime 2. Immediately before meals and at bedtime 3. 30 minutes before meals and at bedtime 4. 1 hour after meals and at bedtime

3. 30 minutes before meals and at bedtime Propantheline bromide is used to reduce secretions and spasms of the GI tract in clients with diverticulitis, a condition characterized by bowel inflammation and colonic irritability and spasticity. The nurse should instruct the client to take the drug 30 minutes before meals and at bedtime to reduce GI motility, thus relieving spasticity. Taking it with a meal, immediately before a meal, or 1 hour after a meal would interfere with the drug's action and absorption, thereby reducing its effectiveness.

Which laboratory value supports a diagnosis of pyelonephritis? 1. Myoglobinuria 2. Ketonuria 3. Pyuria 4. Low white blood cell (WBC) count

3. Pyuria Pyelonephritis is diagnosed by the presence of leukocytosis, hematuria, pyuria, and bacteriuria. The client exhibits fever, chills, and flank pain. Because there is often a septic picture, the WBC count is more likely to be high rather than low, as indicated in option 4. Ketonuria indicates a diabetic state.

Which of the following is an appropriate nursing diagnosis for a client with renal calculi? 1. Ineffective renal tissue perfusion 2. Functional urinary incontinence 3. Risk for infection 4. Decreased cardiac output

3. Risk for infection Infection can occur with renal calculi from urine stasis caused by obstruction. Options 1 and 4 aren't appropriate for this client, and retention of urine, rather than incontinence, usually occurs.

A client comes to the emergency department complaining of acute GI distress. When obtaining the client's history, the nurse inquires about the family history. Which disorder has a familial basis? 1. Hepatitis 2. Iron deficiency anemia 3. Ulcerative colitis 4. Chronic peritonitis

3. Ulcerative colitis Ulcerative colitis is more common in people who have family members with the disease. (The same is true of some types of GI cancers, ulcers, and Crohn's disease.) Hepatitis, iron deficiency anemia, and chronic peritonitis are acquired disorders that don't run in families.

The nurse is caring for a client with acute renal failure. The nurse should expect that hypertonic glucose, insulin infusions, and sodium bicarbonate will be used to treat what complication of acute renal failure? 1. Hypokalemia 2. Hyperphosphatemia 3. Hypophosphatemia 4. Hyperkalemia

4. Hyperkalemia Hyperkalemia is a common complication of acute renal failure. The administration of glucose and regular insulin infusions, with sodium bicarbonate if necessary, can temporarily prevent cardiac arrest by moving potassium into the cells and temporarily reducing potassium levels. This treatment isn't used to treat hyperphosphatemia or hypophosphatemia.

Which medication should the nurse expect to administer to a client with constipation? 1. lorazepam (Ativan) 2. loperamide (Imodium) 3. flurbiprofen (Ansaid) 4. docusate sodium (Colace)

4. docusate sodium (Colace) Docusate sodium, a laxative, is used to treat constipation. It softens the stool by stimulating the secretion of intestinal fluid into the stool. Lorazepam, an antianxiety agent, has no laxative effect. Administering loperamide, an antidiarrheal agent, could cause the constipation to worsen. Flurbiprofen is a nonsteroidal anti-inflammatory agent with no laxative effect.

Chron's disease

An inflammatory disease that can occur anywhere in the gastrointestinal tract but most often affects the terminal ileum and leads to thickening and scarring, a narrowed lumen, fistulas, ulcerations, and abscesses. Characterized by remissions and exacerbations.

Signs of Bowel Perforation and Peritonitis

Guarding of the abdomen Increased fever and chills Pallor Progressive abdominal distention and abdominal pain Restlessness Tachycardia and tachypnea

diverticulitis

Inflammation of one or more diverticula from penetration of fecal matter through the thin-walled diverticula, resulting in local abscess formation. A perforated diverticulum can progress to intra-abdominal perforation with generalized peritonitis.

Interventions for appendicitis

NPO Administer fluids intravenously to prevent dehydration. Monitor for changes in level of pain. Monitor for signs of ruptured appendix and peritonitis Position the client in a right side-lying or low to semi-Fowler's position to promote comfort. Monitor bowel sounds. Apply ice packs to the abdomen for 20 to 30 minutes every hour as prescribed. (Avoid heat - heat can cause rupture) Administer antibiotics as prescribed. Pain meds after dx is made. Schedule appendectomy.

diverticulosis

Outpouching or herniations of the intestinal mucosa that can occur in any part of the intestine but is most common in the sigmoid colon.

Nephrectomy

Performed for extensive kidney damage, renal infection, severe obstruction from stones or tumors, and prevention of stone recurrence.

Pharmacological treatment for BPH

Proscar - long term - blocks testosterone formation causing regression of hyoerplastic tissue and decreases the size of the prostate Minipress, Hytrin, Flomax - relax smooth muscle and increase urinary flow

A client asks a nurse how soon after bowel surgery normal bowel function will return. The best response by the nurse would be: 1. "By 72 hours you should start to pass gas." 2. Around 48 hours, if there are no complications." 3. "Some function will return by 12 hours." 4. "You'll pass gas by 24 hours."

1. "By 72 hours you should start to pass gas." The bowel should be functioning by 72 hours. There may be faint bowel sounds at 48 hours but the bowel will not be fully functional. At 12 hours the effects of general anesthesia on the bowel are still present.

After undergoing a total cystectomy and urinary diversion, a client has a Kock pouch (continent internal reservoir). Which statement by the client indicates a need for further teaching? 1. "I'll have to wear an external collection pouch for the rest of my life." 2. "I should eat foods from all the food groups." 3. "I'll need to drink at least eight glasses of water a day." 4. "I'll have to catheterize my pouch every 2 hours."

1. "I'll have to wear an external collection pouch for the rest of my life." An internal collection pouch, such as the Kock pouch, allows the client to perform self-catheterization for ileal drainage. This pouch is an internal reservoir, eliminating the need for an external collection pouch. A well-balanced diet is essential for healing; the client need not include or exclude particular foods. The client should drink at least eight glasses of fluid daily to prevent calculi formation and urinary tract infection (UTI). Intervals between pouch drainings should be increased gradually until the pouch is emptied two to four times daily.

Interventions for ulcerative colitis

1. Acute phase: Maintain NPO status and administer fluids and electrolytes intravenously or via parenteral nutrition as prescribed. 2. Restrict the client's activity to reduce intestinal activity. 3. Monitor bowel sounds and for abdominal tenderness and cramping. 4. Monitor stools, noting color, consistency, and the presence or absence of blood. 5. Monitor for bowel perforation, peritonitis, and hemorrhage. 6. Following the acute phase, the diet progresses from clear liquids to a low-fiber diet as tolerated. 7. Instruct the client about diet; usually a low-fiber, high-protein diet with vitamins and iron supplements are prescribed. 8. Instruct the client to avoid gas-forming foods, milk products, and foods such as whole wheat grains, nuts, raw fruits and vegetables, pepper, alcohol, and caffeine-containing products. 9. Instruct the client to avoid smoking. 10. Administer medications as prescribed , which may include a combination of medications such as salicylate compounds , corticosteroids, immunosuppressants, and antidiarrheals.

When preparing a client for a hemorrhoidectomy, the nurse should take which action? 1. Administer an enema as ordered. 2. Administer oral antibiotics as prescribed. 3. Administer topical antibiotics as prescribed. 4. Administer analgesics as prescribed.

1. Administer an enema as ordered. When preparing a client for a hemorrhoidectomy, the nurse should administer an enema, as ordered, and record the results. After surgery, the client may require antibiotics and analgesics.

The nurse is teaching a client about the risk factors associated with colorectal cancer. The nurse determines that further teaching related to colorectal cancer is necessary if the client identifies which of the following as an associated risk factor? 1. Age younger than 50 years 2. History of colorectal polyps 3. Family history of colorectal cancer 4. Chronic inflammatory bowel disease

1. Age younger than 50 years Colorectal cancer risk factors include age older than 50 years, a family history of the disease, colorectal polyps, and chronic inflammatory bowel disease.

After undergoing renal arteriogram, in which the left groin was accessed, the client complains of left calf pain. Which intervention should the nurse perform first? 1. Assess peripheral pulses in the left leg. 2. Place cool compresses on the calf. 3. Exercise the leg and foot. 4. Assess for anaphylaxis.

1. Assess peripheral pulses in the left leg. The nurse should begin by assessing peripheral pulses in the left leg to determine if blood flow was interrupted by the procedure. The client may also have thrombophlebitis. Cool compresses aren't used to relieve pain and inflammation in thrombophlebitis. The leg should remain straight after the procedure. Calf pain isn't a symptom of anaphylaxis.

PostOp interventions for rectal surgery

1. Assist the client to a prone or side-lying position to prevent bleeding. 2. Maintain ice packs over the dressing as prescribed until the packing is removed by the physician. 3. Monitor for urinary retention. 4. Administer stool softeners as prescribed. 5. Instruct the client to increase fluids and high-fiber foods. 6. Instruct the client to limit sitting to short periods of time. 7. Instruct the client in the use of sitz baths three or four times a day as prescribed.

A client, age 82, is admitted to an acute care facility for treatment of an acute flare-up of a chronic GI condition. In addition to assessing the client for complications of the current illness, the nurse monitors for age-related changes in the GI tract. Which age-related change increases the risk of anemia? 1. Atrophy of the gastric mucosa 2. Decrease in intestinal flora 3. Increase in bile secretion 4. Dulling of nerve impulses

1. Atrophy of the gastric mucosa Atrophy of the gastric mucosa reduces hydrochloric acid secretion; this, in turn, impairs absorption of iron and vitamin B12, increasing the risk of anemia as a person ages. A decrease in hydrochloric acid increases, not decreases, intestinal flora; as a result, the client is at increased risk for infection, not anemia. A reduction, not increase, in bile secretion may lead to malabsorption of fats and fat-soluble vitamins. Dulling of nerve impulses associated with aging increases the risk of constipation, not anemia.

Interventions for UTIs

1. Before administering prescribed antibiotics, obtain a urine specimen for culture and sensitivity, if prescribed, to identify bacterial growth. 2. Encourage the client to increase fluids up to 3000 mL/ day, especially if the client is taking a sulfonamide; sulfonamides can form crystals in concentrated urine. 3. Administer prescribed medications , which may include analgesics, antiseptics, antispasmodics, antibiotics, and antimicrobials. 4. Maintain an acid urine pH (5.5); instruct the client about foods to consume to maintain acidic urine. 5. Provide heat to the abdomen or sitz baths for complaints of discomfort. 6. Note that if the client is prescribed an aminoglycoside, sulfonamide, or nitrofurantoin (Macrodantin), the actions of these medications are decreased by acidic urine. 7. Use sterile technique when inserting a urinary catheter. 8. Maintain closed urinary drainage systems for the client with an indwelling catheter and avoid elevating the urinary drainage bag above the level of the bladder. 9. Provide meticulous perineal care for the client with an indwelling catheter. 10. Discourage caffeine products such as coffee, tea, and cola. 11. Client education a. Avoid alcohol. b. Take medications as prescribed. c. Take antibiotics on schedule and complete the entire course of medications as prescribed, which may be 10 to 14 days. d. Repeat the urine culture following treatment. e. Prevent recurrence of cystitis

Assessment findings for colon cancer

1. Blood in stool (most common manifestation) 2. Anorexia, vomiting, and weight loss 3. Anemia 4. Abnormal stools a. Ascending colon tumor: Diarrhea b. Descending colon tumor: Constipation or some diarrhea, or flat, ribbon-like stool caused by a partial obstruction c. Rectal tumor: Alternating constipation and diarrhea 5. Guarding or abdominal distention, abdominal mass (late sign) 6. Cachexia (late sign) 7. Masses noted on barium enema, colonoscopy, CT scan, sigmoidoscopy

A client with inflammatory bowel disease undergoes an ileostomy. On the first day after surgery, the nurse notes that the client's stoma appears dusky. How should the nurse interpret this finding? 1. Blood supply to the stoma has been interrupted. 2. This is a normal finding 1 day after surgery. 3. The ostomy bag should be adjusted. 4. An intestinal obstruction has occurred.

1. Blood supply to the stoma has been interrupted. An ileostomy stoma forms as the ileum is brought through the abdominal wall to the surface skin, creating an artificial opening for waste elimination. The stoma should appear cherry red, indicating adequate arterial perfusion. A dusky stoma suggests decreased perfusion, which may result from interruption of the stoma's blood supply and may lead to tissue damage or necrosis. A dusky stoma isn't a normal finding. Adjusting the ostomy bag wouldn't affect stoma color, which depends on blood supply to the area. An intestinal obstruction also wouldn't change stoma color.

A 75-year-old client with renal insufficiency is admitted to the hospital with pneumonia. He's being treated with gentamicin (Garamycin), which can be nephrotoxic. Which laboratory value should be closely monitored? 1. Blood urea nitrogen 2. Sodium level 3. Alkaline phosphatase 4. White blood cell (WBC) count

1. Blood urea nitrogen Blood urea nitrogen and creatinine levels should be closely monitored to detect elevations caused by nephrotoxicity. Sodium level should be routinely monitored in all hospitalized clients. Alkaline phosphatase helps evaluate liver function. The WBC count should be monitored to evaluate the effectiveness of the antibiotic; it doesn't help evaluate kidney function.

Assessment findings for hemorrhoids

1. Bright red bleeding with defecation 2. Rectal pain 3. Rectal itching

Which food should the nurse advise a client with a colostomy to avoid? 1. Carbonated drinks. 2. Fresh-cooked green beans. 3. Liver and bacon. 4. Cooked cereals.

1. Carbonated drinks. Carbonated drinks, cabbage, sauerkraut, and nuts tend to increase flatulence, and most clients feel uncomfortable passing flatus into the colostomy bag, because it causes the bag to inflate. Onions, cheese, and fish may cause odorous drainage.

A client with mild diarrhea, fever, and abdominal discomfort is being evaluated for inflammatory bowel disease (IBD). Which statement about IBD is true? 1. Diarrhea is the most common sign of IBD. 2. Transmural inflammation with fistula formation occurs in ulcerative colitis, one form of IBD. 3. Abscesses may occur in IBD as poor nutrition causes breakdown of cells in the GI tract. 4. Bowel cancer is common in clients with a history of Crohn's disease, one form of IBD.

1. Diarrhea is the most common sign of IBD. IBD is a collective term for several GI inflammatory diseases with unknown causes. The most prominent sign of IBD is mild diarrhea, which sometimes is accompanied by fever and abdominal discomfort. The pathophysiology of ulcerative colitis involves vascular congestion, hemorrhage, and edema — usually affecting the rectum and left colon. Although abscesses may occur in IBD, they result from buildup of lymphocytes and cellular debris in crypts, which may serve as abscess sites. Only about 3% of clients with a long history of Crohn's disease develop bowel cancer.

Assessment findings for BPH

1. Diminished size and force of urinary stream (early sign of BPH) 2. Urinary urgency and frequency 3. Nocturia 4. Inability to start (hesitancy) or continue a urinary stream 5. Feelings of incomplete bladder emptying 6. Postvoid dribbling from overflow incontinence (later sign) 7. Urinary retention and bladder distention 8. Hematuria 9. Urinary stasis 10. Dysuria and bladder pain 11. UTIs

A client returns to the medical-surgical unit after coronary artery bypass graft surgery, which was complicated by prolonged cardiopulmonary bypass and hypotension. After 3 hours in the unit, the client's condition stabilizes. However, the urine output has decreased despite adequate filling pressures. The nurse expects the physician to add which drug, at which flow rate, to the client's regimen? 1. Dopamine (Intropin), 3 mcg/kg/min 2. Epinephrine, 2 mcg/kg/min 3. Dopamine (Intropin), 8 mcg/kg/min 4. Epinephrine, 4 mcg/kg/min

1. Dopamine (Intropin), 3 mcg/kg/min This client is at high risk for acute prerenal failure secondary to decreased renal perfusion during surgery. To dilate the renal arteries and help prevent renal shutdown, the physician is likely to prescribe dopamine at a low flow rate (2 to 5 mcg/kg/min). Although this drug has mixed dopaminergic and beta activity when given at 5 to 10 mcg/kg/min, the client is stabilized and thus doesn't need the beta effects from the higher flow rate — or the sympathomimetic effects of epinephrine. The dopaminergic effects of dopamine increase renal perfusion, contractility, and vasodilation. Stimulation of beta receptors causes beta effects — namely, increases in the heart rate, myocardial contraction force, and cardiac conduction.

The nurse is caring for a client who had a stroke. Which nursing intervention promotes urinary continence? 1. Encouraging intake of at least 2 L of fluid daily 2. Giving the client a glass of soda before bedtime 3. Taking the client to the bathroom twice per day 4. Consulting with a dietitian

1. Encouraging intake of at least 2 L of fluid daily By encouraging a daily fluid intake of at least 2 L, the nurse helps fill the client's bladder, thereby promoting bladder retraining by stimulating the urge to void. The nurse shouldn't give the client soda before bedtime; soda acts as a diuretic and may make the client incontinent. The nurse should take the client to the bathroom or offer the bedpan at least every 2 hours throughout the day; twice per day is insufficient. Consultation with a dietitian won't address the problem of urinary incontinence.

The client is prescribed continuous bladder irrigation at a rate of 60 gtt/minute. The nurse hangs a 2 L bag of sterile solution with tubing on a three-legged I.V. pole. She then attaches the tubing to the client's three-way urinary catheter, adjusts the flow rate, and leaves the room. Which important procedural step did the nurse fail to follow? 1. Evaluating patency of the drainage lumen 2. Counter-balancing the I.V. pole 3. Attaching the infusion set to an infusion pump 4. Collecting a urine specimen before beginning irrigation

1. Evaluating patency of the drainage lumen The nurse should evaluate patency of the drainage tubing before leaving the client's room. If the lumen is obstructed, the solution infuses into the bladder but isn't eliminated through the drainage tubing, a situation that may cause client injury. Balancing the pole is important; however, the nurse would have had to address this issue immediately after hanging the 2 L bag. Using an I.V. pump isn't necessary for continuous bladder irrigation. Unless specifically ordered, obtaining a urine specimen before beginning continuous bladder irrigation isn't necessary.

Assessment findings for Chron's disease

1. Fever 2. Cramp-like and colicky pain after meals 3. Diarrhea (semisolid), which may contain mucus and pus (non-bloody) 4. Abdominal distention 5. Anorexia, nausea, and vomiting 6. Weight loss 7. Anemia 8. Dehydration 9. Electrolyte imbalances 10. Malnutrition (may be worse than that seen in ulcerative colitis)

Assessment findings for UTIs

1. Frequency and urgency 2. Burning on urination 3. Voiding in small amounts 4. Inability to void 5. Incomplete emptying of the bladder 6. Lower abdominal discomfort or back discomfort 7. Cloudy, dark, foul-smelling urine 8. Hematuria 9. Bladder spasms 10. Malaise, chills, fever 11. Nausea and vomiting 12. WBC count greater than 100,000 cells/mm3 on urinalysis 13. An elevated specific gravity and pH may be noted on urinalysis.

Assessment findings for glomerulonephritis

1. Gross hematuria 2. Dark, smoky, cola-colored or red-brown urine 3. Proteinuria that produces a persistent and excessive foam in the urine 4. Urinary debris 5. Moderately elevated to high urine specific gravity 6. Low urinary pH 7. Urinalysis shows large numbers of erythrocytes 8. Oliguria or anuria 9. Headache 10. Chills and fever 11. Fatigue and weakness 12. Anorexia, nausea, and vomiting 13. Pallor 14. Edema in the face, periorbital area, feet, or generalized 15. Shortness of breath, ascites, pleural effusion, and CHF 16. Abdominal or flank pain 17. Hypertension 18. Reduced visual acuity 19. Increased blood urea nitrogen and serum creatinine levels 20. Increased antistreptolysin O titer (used to diagnose disorders caused by streptococcal infections)

The nurse is teaching a client how to irrigate his stoma. Which action indicates that the client needs more teaching? 1. Hanging the irrigation bag 24″ to 36″ (60 to 90 cm) above the stoma 2. Filling the irrigation bag with 500 to 1,000 ml of lukewarm water 3. Stopping irrigation for cramps and clamping the tubing until cramps pass 4. Washing hands with soap and water when finished

1. Hanging the irrigation bag 24″ to 36″ (60 to 90 cm) above the stoma An irrigation bag should be elevated 18″ to 24″ (40 to 60 cm) above the stoma. Typically, adults use 500 to 1,000 ml of water at a temperature no higher than 105° F (41° C) to irrigate a colostomy. If cramping occurs during irrigation, irrigation should be stopped and the client should take deep breaths until the cramping stops. Irrigation can then be resumed. Hand washing reduces the spread of microorganisms.

A client with a retroperitoneal abscess is receiving gentamicin (Garamycin). Which signs should the nurse monitor? 1. Hearing 2. Urine output 3. Hematocrit (HCT) 4. Blood urea nitrogen (BUN) and creatinine levels 5. Serum calcium level

1. Hearing 2. Urine output 4. Blood urea nitrogen (BUN) and creatinine levels Adverse reactions to gentamicin include ototoxicity and nephrotoxicity. The nurse must monitor the client's hearing and instruct him to report any hearing loss or tinnitus. Signs of nephrotoxicity include decreased urine output and elevated BUN and creatinine levels. Gentamicin doesn't affect the serum calcium level or HCT.

If a bowel obstruction occurs from inflammatory bowel disease at the transverse colon, a nurse will initially hear bowel sounds that are: 1. Increased RLQ; decreased LLQ. 2. Decreased RLQ; decreased LLQ. 3. Absent RLQ; absent LLQ. 4. Decreased RLQ; increased LLQ.

1. Increased RLQ; decreased LLQ. Initially peristalsis will increase in the ascending colon (RLQ) in an attempt to clear the blockage. There will be no peristalsis distal to the obstruction.

While a client is being prepared for discharge, the nasogastric (NG) feeding tube becomes clogged. To remedy this problem and teach the client's family how to deal with it at home, what should the nurse do? 1. Irrigate the tube with cola. 2. Advance the tube into the intestine. 3. Apply intermittent suction to the tube. 4. Withdraw the obstruction with a 30-ml syringe.

1. Irrigate the tube with cola. The nurse should irrigate the tube with cola because its effervescence and acidity are suited to the purpose, it's inexpensive, and it's readily available in most homes. Advancing the NG tube is inappropriate because the tube is designed to stay in the stomach and isn't long enough to reach the intestines. Applying intermittent suction or using a syringe for aspiration is unlikely to dislodge the material clogging the tube but may create excess pressure. Intermittent suction may even collapse the tube.

A client comes to the emergency department complaining of sudden onset of sharp, severe pain in the lumbar region, which radiates around the side and toward the bladder. The client also reports nausea and vomiting and appears pale, diaphoretic, and anxious. The physician tentatively diagnoses renal calculi and orders flat-plate abdominal X-rays. Renal calculi can form anywhere in the urinary tract. What is their most common formation site? 1. Kidney 2. Ureter 3. Bladder 4. Urethra

1. Kidney The most common site of renal calculi formation is the kidney. Calculi may travel down the urinary tract with or without causing damage and may lodge anywhere along the tract or may stay within the kidney. The ureter, bladder, and urethra are less common sites of renal calculi formation.

Interventions for pyelonephritis

1. Monitor vital signs, especially for elevated temperature. 2. Encourage fluid intake up to 3000 mL /day to reduce fever and prevent dehydration. 3. Monitor intake and output (ensure that output is a minimum of 1500 mL/ 24 hr). 4. Monitor weight. 5. Encourage adequate rest. 6. Instruct the client in a high-calorie, low-protein diet. 7. Provide warm, moist compresses to the flank area to help relieve pain. 8. Encourage the client to take warm baths for pain relief. 9. Administer analgesics, antipyretics, antibiotics, urinary antiseptics, and antiemetics as prescribed.

Interventions for glomerulonephritis

1. Monitor vital signs, especially for hypertension and temperature elevations. 2. Monitor intake and output and urine characteristics closely. 3. Monitor daily weight. 4. Monitor for edema. 5. Monitor for fluid overload, ascites, pulmonary edema, and CHF. 6. Restrict fluid intake as prescribed. 7. Provide a high-calorie, low-protein, low-sodium, and low-potassium diet to prevent worsening azotemia, fluid retention, and hyperkalemia. 8. Provide bed rest and limit activity. 9. Administer diuretics, antihypertensives, and antibiotics as prescribed. 10. Monitor for signs of renal failure, cardiac failure, and hypertensive encephalopathy. 11. Instruct the client to report signs of bloody urine, headache, or edema. 12. Instruct the client to obtain treatment for infections, especially sore throats, skin lesions, and upper respiratory infections.

Interventions for nephrolithiasis

1. Monitor vital signs, especially the temperature, for signs of infection. 2. Monitor intake and output. 3. Assess for fever, chills, and infection. 4. Monitor for nausea, vomiting, and diarrhea. 5. Encourage fluid intake up to 3000 mL/ day, unless contraindicated, to facilitate the passage of the stone and prevent infection. 6. Administer fluids intravenously as prescribed if unable to take fluids orally or in adequate amounts to increase the flow of urine and facilitate passage of the stone. 7. Provide warm baths and heat to the flank area (massage therapy should be avoided). 8. Administer analgesics at regularly scheduled intervals as prescribed to relieve pain. 9. Assess the client's response to pain medication. 10. Assist the client in performing relaxation techniques to assist in relieving pain. 11. Encourage client ambulation, if stable, to promote the passage of the stone. 12. Turn and reposition the immobilized client to promote passage of the stone. 13. Instruct the client in the diet restrictions specific to the stone composition if prescribed. 14. Prepare the client for surgical procedures if prescribed

PostOp Interventions for urinary diversion

1. Monitor vital signs. 2. Assess incision site. 3. Assess stoma (should be red and moist) every hour for the first 24 hours. 4. Monitor for edema in the stoma, which may be present in the immediate postoperative period. 5. Notify the physician if the stoma appears dark and dusky (indicates necrosis). 6. Monitor for prolapse or retraction of the stoma. 7. Assess bowel function; monitor for expected return of peristalsis in 3 to 4 days. 8. Maintain NPO status as prescribed until bowel sounds return. 9. Monitor continuous urine flow (30 to 60 mL/ hr). 10. Notify the physician if the urine output is less than 30 mL/ hr or if no urine output occurs for more than 15 minutes. 11. Ureteral stents or catheters, if present, may be in place for 2 to 3 weeks or until healing occurs; maintain stability with catheters to prevent dislodgment. 12. Monitor for hematuria. 13. Monitor for signs of peritonitis. 14. Monitor for bladder distention following a partial cystectomy. 15. Monitor for shock, hemorrhage, thrombophlebitis, and lower extremity lymphedema after a radical cystectomy. 16. Monitor the urinary drainage pouch for leaks, and check skin integrity. 17. Monitor the pH of the urine (do not place the dipstick in the stoma) because strongly alkaline urine can cause skin irritation and facilitate crystal formation. 18. Instruct the client regarding the potential for urinary tract infection or the development of calculi. 19. Instruct the client to assess the skin for irritation, monitor the urinary drainage pouch, and report any leakage. 20. Encourage the client to express feelings about changes in body image, embarrassment, and sexual dysfunction.

PostOp prostate surgery

1. Monitor vital signs. 2. Monitor urinary output and urine for hemorrhage or clots. 3. Increase fluids to 2400 to 3000 mL/ day, unless contraindicated. 4. Monitor for arterial bleeding as evidenced by bright red urine with numerous clots; if it occurs, increase CBI and notify the physician immediately. 5. Monitor for venous bleeding as evidenced by burgundy-colored urine output; if it occurs, inform the physician, who may apply traction on the catheter. 6. Monitor hemoglobin and hematocrit levels. 7. Expect red to light pink urine for 24 hours, turning to amber in 3 days. 8. Ambulate the client as early as possible and as soon as urine begins to clear in color. 9. Inform the client that a continuous feeling of an urge to void is normal. 10. Instruct the client to avoid attempts to void around the catheter because this will cause bladder spasms. 11. Administer antibiotics, analgesics, stool softeners, and antispasmodics as prescribed. 12. Monitor the three-way Foley catheter, which usually has a 30- to 45-mL retention balloon. 13. Maintain CBI with sterile bladder irrigation solution as prescribed to keep the catheter free of obstruction and maintain the urine pink in color.

The nurse is monitoring a client admitted to the hospital with a diagnosis of appendicitis who is scheduled for surgery in 2 hours. The client begins to complain of increased abdominal pain and begins to vomit. On assessment, the nurse notes that the abdomen is distended and bowel sounds are diminished. Which is the appropriate nursing intervention? 1. Notify the physician. 2. Administer the prescribed pain medication. 3. Call and ask the operating room team to perform the surgery as soon as possible. 4. Reposition the client and apply a heating pad on warm setting to the client's abdomen.

1. Notify the physician. Based on the signs and symptoms presented in the question, the nurse should suspect peritonitis and notify the physician. Administering pain medication is not an appropriate intervention. Heat should never be applied to the abdomen of a client with suspected appendicitis because of the risk of rupture. Scheduling surgical time is not within the scope of nursing practice, although the physician probably would perform the surgery earlier than the prescheduled time.

The female client is admitted to the emergency department following a fall from a horse and the physician prescribes insertion of a Foley catheter. While preparing for the procedure, the nurse notes blood at the urinary meatus. The nurse should: 1. Notify the physician. 2. Use a small-sized of catheter. 3. Administer pain medication before inserting the catheter. 4. Use extra povidone-iodine solution in cleansing the meatus.

1. Notify the physician. The presence of blood at the urinary meatus may indicate urethral trauma or disruption . The nurse notifies the physician, knowing that the client should not be catheterized until the cause of the bleeding is determined by diagnostic testing. Therefore options 2, 3, and 4 are incorrect.

A client with a history of severe abdominal cramping arrives at the emergency department vomiting a moderate amount of blood. The nurse's primary concern should be: 1. Observing for signs and symptoms of shock. 2. Immediately paging the physician on call. 3. Filling out the appropriate assessment tool. 4. Immediately administering CPR.

1. Observing for signs and symptoms of shock. The client has experienced some gastric bleeding; the client should be observed closely for symptoms of shock. Paging the physician is secondary to the close monitoring of the client's vital signs for symptoms of shock. Completing the appropriate assessment tool may be done later, when the client is under less stress. CPR is not indicated, the client is not experiencing cardiac or respiratory failure.

A client had a positive fecal occult blood test during a health screening. The nursing assessment would include: Select all the apply. 1. Orthostatic blood pressure and pulse. 2. STAT request for hemoglobin and hematocrit. 3. Description of stool volume, color, and consistency. 4. Diet history of raw meat consumption. 5. Regular use of aspirin or NSAIDs. 6. Vitamin C, 250-mg tablets, before the test.

1. Orthostatic blood pressure and pulse. 3. Description of stool volume, color, and consistency. 4. Diet history of raw meat consumption. 5. Regular use of aspirin or NSAIDs. 6. Vitamin C, 250-mg tablets, before the test. Inadequate volume from a significant blood loss will result in a drop of the systolic blood pressure greater than 25 mm Hg and the diastolic value greater than 20 mm Hg, and an increase in pulse rate of 30 bpm when the client goes from flat to stitting/standing. Ordering lab tests is not part of nursing assessment. Nursing assessment would include a description of the characteristics of the stool. Eating red meat before the test may result in a false positive finding. It should be known if the client has a history of frequent and high doses of drugs known to cause GI irritation and bleeding. Taking Vitamin C within 48 hours of a fecal occult blood test will cause a false positive reading.

Two weeks after being diagnosed with a streptococcal infection, a client develops fatigue, a low-grade fever, and shortness of breath. The nurse auscultates bilateral crackles and observes neck vein distention. Urinalysis reveals red and white blood cells and protein. After the physician diagnoses poststreptococcal glomerulonephritis, the client is admitted to the medical-surgical unit. Which immediate action should the nurse take? 1. Place the client on bed rest. 2. Provide a high-protein, fluid-restricted diet. 3. Prepare to assist with insertion of a Tenckhoff catheter for hemodialysis. 4. Place the client on a sheepskin, and monitor for increasing edema.

1. Place the client on bed rest. The nurse immediately must enforce bed rest for a client with glomerulonephritis to ensure a complete recovery and help prevent complications. Depending on disease severity, the client may require fluid, sodium, potassium, and protein restrictions. Because of the risk of altered urinary elimination related to oliguria, this client may require hemodialysis or plasmapheresis for several weeks until renal function improves; however, a Tenckhoff catheter is used in peritoneal dialysis, not hemodialysis. Although comfort measures such as placing the client on a sheepskin are important, they don't take precedence.

The nurse is assessing for stoma prolapse in a client with a colostomy. What should the nurse observe if stoma prolapse occurs? 1. Protruding stoma 2. Sunken and hidden stoma 3. Narrowed and flattened stoma 4. Dark- and bluish-colored stoma

1. Protruding stoma A prolapsed stoma is one in which the bowel protrudes through the stoma. A stoma retraction is characterized by sinking of the stoma. Ischemia of the stoma would be associated with a dusky or bluish color. A stoma with a narrowed opening at the level of the skin or fascia is said to be stenosed.

Interventions for diverticulitis

1. Provide bedrest during the acute phase. 2. Maintain NPO status or provide clear liquids during the acute phase as prescribed. 3. Introduce a fiber-containing diet gradually, when the inflammation has resolved. 4. Administer antibiotics, analgesics, and anticholinergics to reduce bowel spasms as prescribed. 5. Instruct the client to refrain from lifting, straining, coughing, or bending to avoid increased intra-abdominal pressure. 6. Monitor for perforation, hemorrhage, fistulas, and abscesses. 7. Instruct the client to increase fluid intake to 2500 to 3000 mL daily, unless contraindicated. 8. Instruct the client to eat soft high-fiber foods, such as whole grains; the client should avoid high-fiber foods when inflammation occurs because these foods will irritate the mucosa further. 9. Instruct the client to avoid gas-forming foods or foods containing indigestible roughage, seeds, or nuts because these food substances become trapped in diverticula and cause inflammation. 10. Instruct the client to consume a small amount of bran daily and to take bulk-forming laxatives as prescribed to increase stool mass.

Assessment findings for nephrolithiasis

1. Renal colic, which originates in the lumbar region and radiates around the side and down to the testicles in men and to the bladder in women 2. Ureteral colic, which radiates toward the genitalia and thighs 3. Sharp, severe pain of sudden onset 4. Dull, aching pain in the kidney 5. Nausea and vomiting, pallor, and diaphoresis during acute pain 6. Urinary frequency, with alternating retention 7. Signs of a urinary tract infection 8. Low-grade fever 9. High numbers of red blood cells, white blood cells, and bacteria noted in the urinalysis report 10. Gross hematuria

The nurse is developing a discharge plan for a client with a new ileostomy. The client is 32 years old, mentally competent, lives with her husband of 3 years, and works outside her home. Which nursing diagnoses are important to include in the discharge plan? Select all that apply. 1. Risk for fluid volume deficit. 2. Risk for fluid volume excess. 3. Risk for constipation. 4. Risk for impaired skin integrity. 5. Altered nutrition: more than body requirements. 6. Altered sexuality patterns.

1. Risk for fluid volume deficit. 4. Risk for impaired skin integrity. 6. Altered sexuality patterns. An ileostomy produced liquid stool. The client's daily fluid intake should be 2 to 3 liters to prevent dehydration. Fecal drainage or a poorly fitting ostomy appliance can irritate and erode the stoma and surrounding the skin. A client with an ileostomy is more likely at risk for nutrition less than body requirements. The client with an ileostomy is often concerned about loss of sexual appeal or leakage of fecal material during sexual activity.

The nurse suspects wound dehiscence, and lifts the edges of the client's dressings. The nurse notes that the wound edges are entirely separated. What is the next nursing action? 1. Tell the client to remain quiet and not to cough. 2. Offer the client a warm drink to promote relaxation. 3. Position the cient in a chair with the feet elevated. 4. Apply a Scultetus bandage.

1. Tell the client to remain quiet and not to cough. The client should remain quiet in a low Fowler's or horizontal position. They should be cautioned not to cough so as note to extrutde any intestines by increasing intra-abdominal pressures. The physician should be notified next. Remain with the client, reassuring them, monitoring vital signs, and having others bring equipment such as IV setup, nasogastric tube, and suction equipment. The client should be kept NPO. The dressing should be left in place to prevent evisceration.

The graduate nurse and her preceptor are establishing priorities for their morning assessments. Which client should they assess first? 1. The newly admitted client with acute abdominal pain 2. The client who underwent surgery three days ago and who now requires a dressing change 3. The client receiving continuous tube feedings who needs the tube-feeding residual checked 4. The sleeping client who received pain medication 1 hour ago

1. The newly admitted client with acute abdominal pain The graduate nurse and her preceptor should assess the new admission with acute abdominal pain first because he just arrived on the floor and might be unstable. Next, they should change the abdominal dressing for the postoperative client or measure feeding tube residual in the client with continuous tube feedings. These tasks are of equal importance. They should assess the sleeping client who received pain medication 1 hour ago last because he just received relief from his pain and is able to sleep.

The client had a new colostomy created 2 days earlier and is beginning to pass malodorous flatus from the stoma. What is the correct interpretation by the nurse? 1. This is a normal, expected event. 2. The client is experiencing early signs of ischemic bowel. 3. The client should not have the nasogastric tube removed. 4. This indicates inadequate preoperative bowel preparation.

1. This is a normal, expected event. As peristalsis returns following creation of a colostomy, the client begins to pass malodorous flatus. This indicates returning bowel function and is an expected event. Within 72 hours of surgery, the client should begin passing stool via the colostomy. Options 2, 3, and 4 are incorrect.

A client with benign prostatic hyperplasia doesn't respond to medical treatment and is admitted to the facility for prostate gland removal. Before providing preoperative and postoperative instructions to the client, the nurse asks the surgeon which prostatectomy procedure will be done. What is the most widely used procedure for prostate gland removal? 1. Transurethral resection of the prostate (TURP) 2. Suprapubic prostatectomy 3. Retropubic prostatectomy 4. Transurethral laser incision of the prostate

1. Transurethral resection of the prostate (TURP) TURP is the most widely used procedure for prostate gland removal. Because it requires no incision, TURP is especially suitable for men with relatively minor prostatic enlargements and for those who are poor surgical risks. Suprapubic prostatectomy, retropubic prostatectomy, and transurethral laser incision of the prostate are less common procedures; they all require an incision.

A client had a nephrectomy 2 days ago and is now complaining of abdominal pressure and nausea. The first nursing action should be to: 1. auscultate bowel sounds. 2. palpate the abdomen. 3. change the client's position. 4. insert a rectal tube.

1. auscultate bowel sounds. If abdominal distention is accompanied by nausea, the nurse must first auscultate bowel sounds. If bowel sounds are absent, the nurse should suspect gastric or small intestine dilation and these findings must be reported to the physician. Palpation should be avoided postoperatively with abdominal distention. If peristalsis is absent, changing positions and inserting a rectal tube won't relieve the client's discomfort.

A triple-lumen indwelling urinary catheter is inserted for continuous bladder irrigation following a transurethral resection of the prostate. In addition to balloon inflation, the functions of the three lumens include: 1. continuous inflow and outflow of irrigation solution. 2. intermittent inflow and continuous outflow of irrigation solution. 3. continuous inflow and intermittent outflow of irrigation solution. 4. intermittent flow of irrigation solution and prevention of hemorrhage.

1. continuous inflow and outflow of irrigation solution. When preparing for continuous bladder irrigation, a triple-lumen indwelling urinary catheter is inserted. The three lumens provide for balloon inflation and continuous inflow and outflow of irrigation solution.

The nurse must provide total parenteral nutrition (TPN) to a client through a triple-lumen central line. To prevent complications of TPN, the nurse should: 1. cover the catheter insertion site with an occlusive dressing. 2. use clean technique when changing the dressing. 3. insert an indwelling urinary catheter. 4. keep the client on complete bed rest.

1. cover the catheter insertion site with an occlusive dressing. TPN increases the client's risk of infection because the catheter insertion site creates a port of entry for bacteria. To reduce the risk of infection, the nurse should cover the insertion site with an occlusive dressing, which is airtight. Because the insertion site is an open wound, the nurse should use sterile technique when changing the dressing. TPN doesn't necessitate placement of an indwelling urinary catheter or bed rest.

The nurse is caring for a client with a colostomy. The client tells the nurse that he makes small pin holes in the drainage bag to help relieve gas. The nurse should teach him that this action: 1. destroys the odor-proof seal. 2. won't affect the colostomy system. 3. is appropriate for relieving the gas in a colostomy system. 4. destroys the moisture-barrier seal.

1. destroys the odor-proof seal. Any hole, no matter how small, will destroy the odor-proof seal of a drainage bag. Removing the bag or unclamping it is the only appropriate method for relieving gas.

A client is admitted for treatment of glomerulonephritis. On initial assessment, the nurse detects one of the classic signs of acute glomerulonephritis of sudden onset. Such signs include: 1. generalized edema, especially of the face and periorbital area. 2. green-tinged urine. 3. moderate to severe hypotension. 4. polyuria.

1. generalized edema, especially of the face and periorbital area. Generalized edema, especially of the face and periorbital area, is a classic sign of acute glomerulonephritis of sudden onset. Other classic signs and symptoms of this disorder include hematuria (not green-tinged urine), proteinuria, fever, chills, weakness, pallor, anorexia, nausea, and vomiting. The client also may have moderate to severe hypertension (not hypotension), oliguria or anuria (not polyuria), headache, reduced visual acuity, and abdominal or flank pain.

A client with severe inflammatory bowel disease is receiving total parenteral nutrition (TPN). When administering TPN, the nurse must take care to maintain the prescribed flow rate because giving TPN too rapidly may cause: 1. hyperglycemia. 2. air embolism. 3. constipation. 4. dumping syndrome.

1. hyperglycemia. Hyperglycemia may occur if TPN is administered too rapidly, exceeding the client's glucose metabolism rate. With hyperglycemia, the renal threshold for glucose reabsorption is exceeded and osmotic diuresis occurs, leading to dehydration and electrolyte depletion. Although air embolism may occur during TPN administration, this problem results from faulty catheter insertion, not overly rapid administration. TPN may cause diarrhea, not constipation, especially if administered too rapidly. Dumping syndrome results from food moving through the GI tract too quickly; because TPN is given I.V., it can't cause dumping syndrome.

A client is recovering from an ileostomy that was performed to treat inflammatory bowel disease. During discharge teaching, the nurse should stress the importance of: 1. increasing fluid intake to prevent dehydration. 2. wearing an appliance pouch only at bedtime. 3. consuming a low-protein, high-fiber diet. 4. taking only enteric-coated medications.

1. increasing fluid intake to prevent dehydration. Because stool forms in the large intestine, an ileostomy typically drains liquid waste. To avoid fluid loss through ileostomy drainage, the nurse should instruct the client to increase fluid intake. The nurse should teach the client to wear a collection appliance at all times because ileostomy drainage is incontinent, to avoid high-fiber foods because they may irritate the intestines, and to avoid enteric-coated medications because the body can't absorb them after an ileostomy.

A client with amebiasis, an intestinal infection, is prescribed metronidazole (Flagyl). When teaching the client about adverse reactions to this drug, the nurse should mention: 1. metallic taste. 2. tinnitus. 3. blurred vision. 4. loss of smell.

1. metallic taste. Metronidazole commonly causes a metallic taste. Other adverse reactions include nausea, anorexia, headache, and dry mouth. The drug isn't associated with tinnitus, blurred vision, or loss of smell.

A 32-year-old male client with appendicitis is experiencing excruciating abdominal pain. An abdominal X-ray film reveals intraperitoneal air. The nurse should prepare the client for: 1. surgery. 2. colonoscopy. 3. nasogastric tube insertion. 4. barium enema.

1. surgery. The client should be prepared for surgery because his signs and symptoms indicate bowel perforation. Appendicitis is the most common cause of bowel perforation in the United States. Because perforation can lead to peritonitis and sepsis, surgery wouldn't be delayed to perform any other intervention. Also, none of the other procedures are necessary at this point.

A client is diagnosed with Crohn's disease after undergoing two weeks of testing. The client's boss calls the medical-surgical floor requesting to speak with the nurse manager. He expresses concern over the client and explains that he must know the client's diagnosis for insurance purposes. Which response by the nurse is best? 1. "Sure, I understand how demanding insurance companies can be." 2. "I appreciate your concern, but I can't give out any information." 3. "Why don't you come in, and we can further discuss this issue." 4. "He has been diagnosed with Crohn's Disease."

2. "I appreciate your concern, but I can't give out any information." The nurse may not release any confidential information to unauthorized individuals, such as the client's boss. Options 1, 3, and 4 breech client confidentiality.

The client understands post-ileostomy care when the client states: 1. "I empty my bag when it gets too heavy." 2. "I empty my bag before it is half full." 3. "I empty my bag before it is one-third full." 4. I don't need to wear a bag all the time."

2. "I empty my bag before it is half full." The bag should be emptied before it gets "too heavy." Emptying the bag when it is about half full will prevent problems with the bag falling off from the weight of the effluent. Emptying the bag when it is only one-third full could be expensive because they will need to buy supplies more frequently. A client with an ileostomy needs to wear a bag all the time.

The nurse is teaching an elderly client about good bowel habits. Which statement by the client would indicate to the nurse that additional teaching is required? 1. "I should eat a fiber-rich diet with raw, leafy vegetables, unpeeled fruit, and whole grain bread." 2. "I need to use laxatives regularly to prevent constipation." 3. "I need to drink 2 to 3 liters of fluid every day." 4. "I should exercise four times per week."

2. "I need to use laxatives regularly to prevent constipation." The elderly client should be taught to gradually eliminate the use of laxatives. Point out that using laxatives to promote regular bowel movements may have the opposite effect. A high-fiber diet, ample amounts of fluids, and regular exercise promote good bowel health.

A client undergoes extracorporeal shock wave lithotripsy. Before discharge, the nurse should provide which instruction? 1. "Take your temperature every 4 hours." 2. "Increase your fluid intake to 2 to 3 L per day." 3. "Apply an antibacterial dressing to the incision daily." 4. "Be aware that your urine will be cherry-red for 5 to 7 days."

2. "Increase your fluid intake to 2 to 3 L per day." Increasing fluid intake flushes the renal calculi fragments through — and prevents obstruction of — the urinary system. Measuring temperature every 4 hours isn't needed. Lithotripsy doesn't require an incision. Hematuria may occur for a few hours after lithotripsy but should then disappear.

A client is recovering from a small-bowel resection. To relieve pain, the physician prescribes meperidine (Demerol), 75 mg I.M. every 4 hours. How soon after administration should meperidine's onset of action occur? 1. 5 to 10 minutes 2. 10 to 15 minutes 3. 30 to 60 minutes 4. 2 to 4 hours

2. 10 to 15 minutes Meperidine's onset of action is 10 to 15 minutes. It peaks between 30 and 60 minutes and has a duration of action of 2 to 4 hours.

After admission for acute appendicitis, a client undergoes an appendectomy. He complains of moderate postsurgical pain for which the physician prescribes pentazocine (Talwin), 50 mg by mouth every 4 hours. How soon after administration of this drug can the nurse expect the client to feel relief? 1. Less than 15 minutes 2. 15 to 30 minutes 3. 30 to 60 minutes 4. 1 to 2 hours

2. 15 to 30 minutes Orally administered pentazocine has an onset of action of 15 to 30 minutes, reaches peak concentration in less than 1 hour, and has a duration of 3 to 4 hours.

The nurse knows the colostomy begins functioning: 1. Immediately. 2. 2 to 3 days postoperatively. 3. 1 week postoperatively. 4. 2 weeks postoperatively.

2. 2 to 3 days postoperatively. Ileostomies begin to drain immediately. The stoma will begin to secrete mucus within 48 hours, and the proximal loop should begin to drain fecal material within 72 hours.

The nurse is monitoring a client receiving paregoric to treat diarrhea for drug interactions. Which drugs can produce additive constipation when given with an opium preparation? 1. Antiarrhythmic drugs 2. Anticholinergic drugs 3. Anticoagulant drugs 4. Antihypertensive drugs

2. Anticholinergic drugs Paregoric has an additive effect of constipation when used with anticholinergic drugs. Antiarrhythmics, anticoagulants, and antihypertensives aren't known to interact with paregoric.

Which task can the nurse delegate to a nursing assistant? 1. Irrigating a nasogastric (NG) tube 2. Assisting a client who had surgery three days ago walk down the hallway 3. Helping a client who just returned from surgery to the bathroom 4. Administering an antacid to a client complaining of heartburn

2. Assisting a client who had surgery three days ago walk down the hallway Because the client had surgery three days ago, the nurse can safely delegate the task of helping the client walk down the hallway. Irrigating an NG tube, administering medications, and assisting a client who just returned from surgery are tasks that must be performed by licensed nursing personnel.

The client with a new ileostomy has received discharge teaching. Which dinner menu selected by the client indicates understanding of an appropriate diet for a new ileostomy? 1. Baked chicken, whole-grain biscuit, corn on the cob, canned peaches. 2. Baked chicken, mashed potato, cooked carrots, angel food cake. 3. Ham, mashed potato, salad with raw carrots, canned peaches. 4. Roast beef, pasta with butter and garlic, split pea soup, 2 chocolate chip cookies with walnuts.

2. Baked chicken, mashed potato, cooked carrots, angel food cake. Whole grains, corn, peas, nuts, and raw vegetables are considered high-fiber foods and may cause blockages. Garlic may produce odor.

A 72-year-old client seeks help for chronic constipation. This is a common problem for elderly clients due to several factors related to aging. Which is one such factor? 1. Increased intestinal motility 2. Decreased abdominal strength 3. Increased intestinal bacteria 4. Decreased production of hydrochloric acid

2. Decreased abdominal strength Decreased abdominal strength, muscle tone of the intestinal wall, and motility all contribute to chronic constipation in the elderly. A decrease in hydrochloric acid causes a decrease in absorption of iron and B12, whereas an increase in intestinal bacteria actually causes diarrhea.

The nurse is reviewing the client's record and notes that the physician has documented that the client has a renal disorder. On review of the laboratory results, the nurse most likely would expect to note which of the following? 1. Decreased hemoglobin level 2. Elevated creatinine level 3. Decreased red blood cell count 4. Decreased white blood cell count

2. Elevated creatinine level Measuring the creatinine level is a frequently used laboratory test to determine renal function. The creatinine level increases when at least 50% of renal function is lost. A decreased hemoglobin level and red blood cell count may be noted if bleeding from the urinary tract occurs or if erythropoietic function by the kidney is impaired. An increased white blood cell count is most likely to be noted in renal disease.

The nurse asks a client who had abdominal surgery 3 days ago if he has moved his bowels since surgery. The client states, "I haven't moved my bowels, but I am passing gas." How should the nurse intervene? 1. Apply moist heat to the client's abdomen. 2. Encourage the client to ambulate at least three times per day. 3. Administer a tap water enema. 4. Notify the physician.

2. Encourage the client to ambulate at least three times per day. The nurse should encourage the client to ambulate at least three times per day. Ambulating stimulates peristalsis, which helps the bowels to move. It isn't appropriate to apply heat to a surgical wound. Moreover, heat application can't be initiated without a physician order. A tap water enema is typically administered as a last resort after other methods fail. A physician's order is needed with a tap water enema as well. Notifying the physician isn't necessary at this point because the client is exhibiting bowel function by passing flatus.

The nurse is reviewing a client's fluid intake and output record. Fluid intake and urine output should relate in which way? 1. Fluid intake should be double the urine output. 2. Fluid intake should be approximately equal to the urine output. 3. Fluid intake should be half the urine output. 4. Fluid intake should be inversely proportional to the urine output.

2. Fluid intake should be approximately equal to the urine output. Normally, fluid intake is approximately equal to the urine output. Any other relationship signals an abnormality. For example, fluid intake that is double the urine output indicates fluid retention; fluid intake that is half the urine output indicates dehydration. Normally, fluid intake isn't inversely proportional to the urine output.

The nurse is reviewing the history of a client with bladder cancer. The nurse expects to note documentation of which most common symptom of this type of cancer? 1. Dysuria 2. Hematuria 3. Urgency on urination 4. Frequency of urination

2. Hematuria The most common symptom in clients with cancer of the bladder is hematuria. The client also may experience irritative voiding symptoms such as frequency, urgency, and dysuria, and these symptoms often are associated with carcinoma in situ.

The nurse is assessing a client who is receiving total parenteral nutrition (TPN). Which finding suggests that the client has developed hyperglycemia? 1. Cheyne-Stokes respirations 2. Increased urine output 3. Decreased appetite 4. Diaphoresis

2. Increased urine output Glucose supplies most of the calories in TPN; if the glucose infusion rate exceeds the client's rate of glucose metabolism, hyperglycemia arises. When the renal threshold for glucose reabsorption is exceeded, osmotic diuresis occurs, causing an increased urine output. A decreased appetite and diaphoresis suggest hypoglycemia. Cheyne-Stokes respirations are characterized by a period of apnea lasting 10 to 60 seconds, followed by gradually increasing depth and frequency of respirations. Cheyne-Stokes respirations typically occur with cerebral depression or heart failure.

The nurse is caring for a client with acute pyelonephritis. Which nursing intervention is most important? 1. Administering a sitz bath twice per day 2. Increasing fluid intake to 3 L/day 3. Using an indwelling urinary catheter to measure urine output accurately 4. Encouraging the client to drink cranberry juice to acidify the urine

2. Increasing fluid intake to 3 L/day Acute pyelonephritis is a sudden inflammation of the interstitial tissue and renal pelvis of one or both kidneys. Infecting bacteria are normal intestinal and fecal flora that grow readily in urine. Pyelonephritis may result from procedures that involve the use of instruments (such as catheterization, cystoscopy, and urologic surgery) or from hematogenic infection. The most important nursing intervention is to increase fluid intake to 3 L/day. This helps empty the bladder of contaminated urine and prevents calculus formation. Administering a sitz bath would increase the likelihood of fecal contamination. Using an indwelling urinary catheter could cause further contamination. Encouraging the client to drink cranberry juice to acidify urine is helpful but isn't the most important interaction.

The nurse is assessing a client who complains of abdominal pain, nausea, and diarrhea. When examining the client's abdomen, which sequence should the nurse use? 1. Inspection, palpation, percussion, and auscultation 2. Inspection, auscultation, percussion, and palpation 3. Auscultation, inspection, percussion, and palpation 4. Palpation, auscultation, percussion, and inspection

2. Inspection, auscultation, percussion, and palpation The correct sequence for abdominal examination is inspection, auscultation, percussion, and palpation. This sequence differs from that used for other body regions (inspection, palpation, percussion, and auscultation) because palpation and percussion increase intestinal activity, altering bowel sounds. Therefore, the nurse shouldn't palpate or percuss the abdomen before auscultating. Assessment of any body system or region starts with inspection; therefore, auscultating or palpating the abdomen first would be incorrect.

The client has a NG tube connected to low continuous suction for abdominal decompression. The nurse notes that gastric fluid in the suction tubing is not moving and the client's abdomen is becoming distended. The nurse's best action is to: 1. Pull out the NG tube and insert a new one. 2. Irrigate the tube with 30 mL of water. 3. Tell the client to take a few deep breaths. 4. Turn the suction higher.

2. Irrigate the tube with 30 mL of water. Problem solving should be done first. Inserting a new NG tube would cause the client unnecessary discomfort. The mos likely cause of the problem is that the NG tube is plugged with gastric contents of has adhered to the gastric mucosa and is no longer draining. Irrigating the tube should clear any obstructions and free the tube from the gastric mucosa. Having the client take a few deep breaths will not solve the problem. Turning the suction higher may cause additional trauma to the gastric mucosa.

A nurse is observing a client for possible complications of postoperative peritonitis. Which manifestations are most indicative of peritonitis? Select all that apply. 1. Hyperactive bowel sounds. 2. Localized or diffuse pain. 3. Abdominal rigidity. 4. Shallow respirations. 5. Temperature over 102.

2. Localized or diffuse pain. 3. Abdominal rigidity. 4. Shallow respirations. Peristalsis is diminished and paralytic ileus may develop. Pain is often diffuse initially and then localized. The abdominal muscles are rigid and the abdomen is tender and distended. With abdominal tenderness and distension, the client will not breathe as deeply. Temperature is usually 100 - 101.

The most important postoperative nursing concern for a client following an ileostomy is to: 1. Check for the presence of diarrhea. 2. Maintain skin-protective barrier. 3. Allow the client to observe irrigation. 4. Change the stoma pouch daily when full.

2. Maintain skin-protective barrier. The drainage from an ileostomy will always be liquid. The first concern is the stoma and the condition of the skin around the stoma. The drainage from the stoma is made up of digestive enzymes. The pouch opening should be no more than 1/8 inch larger than the stoma. Irrigation is not done with an ileostomy. The pouch is empties, not changed, when it is 1/3 full. It is changed every 5-7 days, usually before eating, when the stoma is least active.

A client with severe abdominal pain is being evaluated for appendicitis. What is the most common cause of appendicitis? 1. Rupture of the appendix 2. Obstruction of the appendix 3. A high-fat diet 4. A duodenal ulcer

2. Obstruction of the appendix Appendicitis most commonly results from obstruction of the appendix, which may lead to rupture. A high-fat diet or duodenal ulcer doesn't cause appendicitis; however, a client may require dietary restrictions after an appendectomy.

When preparing a client, age 50, for surgery to treat appendicitis, the nurse formulates a nursing diagnosis of Risk for infection related to inflammation, perforation, and surgery. What is the rationale for choosing this nursing diagnosis? 1. Obstruction of the appendix may increase venous drainage and cause the appendix to rupture. 2. Obstruction of the appendix reduces arterial flow, leading to ischemia, inflammation, and rupture of the appendix. 3. The appendix may develop gangrene and rupture, especially in a middle-aged client. 4. Infection of the appendix diminishes necrotic arterial blood flow and increases venous drainage.

2. Obstruction of the appendix reduces arterial flow, leading to ischemia, inflammation, and rupture of the appendix. A client with appendicitis is at risk for infection related to inflammation, perforation, and surgery because obstruction of the appendix causes mucus fluid to build up, increasing pressure in the appendix and compressing venous outflow drainage. The pressure continues to rise with venous obstruction; arterial blood flow then decreases, leading to ischemia from lack of perfusion. Inflammation and bacterial growth follow, and swelling continues to raise pressure within the appendix, resulting in gangrene and rupture. Geriatric, not middle-aged, clients are especially susceptible to appendix rupture.

The client returns to the postanesthesia care unit following GI surgery. Postoperative laboratory values include: O2 saturation 99%, WBC 11,500, Hct 35, Na+ 139, K+ 3.9, glucose 51. The first priority of the nurse would be to: 1. Assess the postoperative wound for infection. 2. Obtain an order for dextrose solution IV. 3. Increase nasal O2 to improve oxygen delivery. 4. Decrease amount of K+ in IV to avoid arrhythmias.

2. Obtain an order for dextrose solution IV. It is too soon for any indication of wound infection. The slight elevation in the WBC count is most likely related to the reason for the GI surgery. The client's serum glucose level is dangerously low and will interfere with accurately assessing the level of consciousness as the client is waking up from anesthesia. Glucose is needed for normal brain functioning. The lowest normal for serum glucose is 70 mg/dL. The blood is well saturated with oxygen. The potassium level is low normal.

What nursing action best facilitates the passage of the NG tube from the stomach though the pylorous and into the duodenum? 1. Gently advancing the tube 1 to 4 inches at the regular time intervals. 2. Positioning the client on the right side for 2 hours after insertion. 3. Maintaining strict bedrest and avoiding all unnecessary movement. 4. Positioning the client in a flat supine position.

2. Positioning the client on the right side for 2 hours after insertion.

Following surgery for a ruptured appendix, the nurse should place the client in a semi-Fowler's position primarily to: 1. Fully aerate the lungs. 2. Promote drainage and prevent subdiaphragmatic abscesses. 3. Splint the wound. 4. Facilitate movement and reduce complications from immobility.

2. Promote drainage and prevent subdiaphragmatic abscesses. Although this position will help aerate the lungs by allowing for maximum expansion, it is not he primary reason for this position. This position will no help splint the incision. This position will not help facilitate movement or necessarily reduce complications.

One day after surgery for intestinal resection, a client has no bowel sounds. Which action should a nurse take? 1. Take the vital signs and notify the physician. 2. Record this expected finding. 3. Check rectally for impacted stool. 4. Perform abdominal massage.

2. Record this expected finding. Paralytic ileus and the absence of bowel sounds is expected on the first postop day.

A client is scheduled for bowel resection with anastomosis involving the large intestine. Because of the surgical site, the nurse formulates the nursing diagnosis of Risk for infection. To complete the nursing diagnosis statement, the nurse should add which "related-to" phrase? 1. Related to major surgery required by bowel resection 2. Related to the presence of bacteria at the surgical site 3. Related to malnutrition secondary to bowel resection with anastomosis 4. Related to the presence of a nasogastric (NG) tube postoperatively

2. Related to the presence of bacteria at the surgical site The large intestine normally contains bacteria because its alkaline environment permits growth of organisms that putrefy and break down remaining proteins and indigestible residue. These organisms include Escherichia coli, Aerobacter aerogenes, Clostridium perfringens, and Lactobacillus. Although bowel resection with anastomosis is considered major surgery, it poses no greater risk of infection than any other type of major surgery. Malnutrition seldom follows bowel resection with anastomosis because nutritional absorption (except for some water, sodium, and chloride) is completed in the small intestine. An NG tube is placed through a natural opening, not a wound, and therefore doesn't increase the client's risk of infection.

The day after colostomy surgery an adult says to the nurse, "I know the doctor did not really do a colostomy." The nurse understands that he is in an early stage of adjustment to his diagnosis and surgery. What nursing action is indicated at this time? 1. Agree with him until he is ready to accept his colostomy. 2. Say "It must be difficult to have this kind of surgery." 3. Force him to look at his colostomy. 4. Ask the surgeon to explain the surgery to the client.

2. Say "It must be difficult to have this kind of surgery."

The client is admitted to the emergency department following a motor vehicle accident. The client was wearing a lap seat belt when the accident occurred and now the client has hematuria and lower abdominal pain. To assess further whether the pain is caused by bladder trauma, the nurse asks the client if the pain is referred to which of the following areas? 1. Hip 2. Shoulder 3. Umbilicus 4. Costovertebral angle

2. Shoulder Bladder trauma or injury is characterized by lower abdominal pain that may radiate to one of the shoulders due to phrenic nerve irritation. Bladder injury pain does not radiate to the umbilicus, costovertebral angle, or hip.

The nurse can quickly assess volume depletion in a client with ulcerative colitits by: 1. Measuring the quantity and specific gravity of the client's urine output. 2. Taking the client's blood pressure first supine, then sitting, noting any changes. 3. Comparing the client's present weight with the weight on a previous admission. 4. Administering the oral water test.

2. Taking the client's blood pressure first supine, then sitting, noting any changes. Urine output and specific gravity are better measures of the adequacy of fluid volume replacement than of fluid volume depletion. Postural blood pressure readings are an excellent mode for assessing volume depletion. If the systolic blood pressure decreases more than 10 mm Hg and there is a concurrent increase in pulse rate, a volume depletion problem is indicated.

When a client with an indwelling urinary catheter insists on walking to the hospital lobby to visit with family members, the nurse teaches him how to do this without compromising the catheter. Which client action indicates an accurate understanding of this information? 1. The client sets the drainage bag on the floor while sitting down. 2. The client keeps the drainage bag below the bladder at all times. 3. The client clamps the catheter drainage tubing while visiting with the family. 4. The client loops the drainage tubing below its point of entry into the drainage bag.

2. The client keeps the drainage bag below the bladder at all times. To maintain effective drainage, the client should keep the drainage bag below the bladder; this allows the urine to flow by gravity from the bladder to the drainage bag. The client shouldn't lay the drainage bag on the floor because it could become grossly contaminated. The client shouldn't clamp the catheter drainage tubing because this impedes the flow of urine. To promote drainage, the client may loop the drainage tubing above — not below — its point of entry into the drainage bag.

After having transurethral resection of the prostate (TURP), a client returns to the unit with a three-way indwelling urinary catheter and continuous closed bladder irrigation. Which finding suggests that the client's catheter is occluded? 1. The urine in the drainage bag appears red to pink. 2. The client reports bladder spasms and the urge to void. 3. The normal saline irrigant is infusing at a rate of 50 drops/min. 4. About 1,000 ml of irrigant have been instilled; 1,200 ml of drainage have been returned.

2. The client reports bladder spasms and the urge to void. Reports of bladder spasms and the urge to void suggest that a blood clot may be occluding the catheter. After TURP, urine normally appears red to pink, and normal saline irrigant usually is infused at a rate of 40 to 60 drops/min or according to facility protocol. The amount of returned fluid (1,200 ml) should correspond to the amount of instilled fluid, plus the client's urine output (1,000 ml + 200 ml), which reflects catheter patency.

The nurse is assessing the colostomy of a client who has had an abdominal perineal resection for a bowel tumor. Which of the following assessment findings indicates that the colostomy is beginning to function? 1. Absent bowel sounds 2. The passage of flatus 3. The client's ability to tolerate food 4. Bloody drainage from the colostomy

2. The passage of flatus Following abdominal perineal resection, the nurse would expect the colostomy to begin to function within 72 hours after surgery, although it may take up to 5 days . The nurse should assess for a return of peristalsis, listen for bowel sounds, and check for the passage of flatus . Absent bowel sounds would not indicate the return of peristalsis. The client would remain NPO until bowel sounds return and the colostomy is functioning. Bloody drainage is not expected from a colostomy.

A client with bladder cancer has had the bladder removed and an ileal conduit created for urine diversion. While changing this client's pouch, the nurse observes that the area around the stoma is red, weeping, and painful. What should the nurse conclude? 1. The skin wasn't lubricated before the pouch was applied. 2. The pouch faceplate doesn't fit the stoma. 3. A skin barrier was applied properly. 4. Stoma dilation wasn't performed.

2. The pouch faceplate doesn't fit the stoma. If the pouch faceplate doesn't fit the stoma properly, the skin around the stoma will be exposed to continuous urine flow from the stoma, causing excoriation and red, weeping, and painful skin. A lubricant shouldn't be used because it would prevent the pouch from adhering to the skin. When properly applied, a skin barrier prevents skin excoriation. Stoma dilation isn't performed with an ileal conduit, although it may be done with a colostomy if ordered.

During rectal examination, which finding would be further evidence of a urethral injury? 1. A low-riding prostate 2. The presence of a boggy mass 3. Absent sphincter tone 4. A positive Hemoccult

2. The presence of a boggy mass When the urethra is ruptured, a hematoma or collection of blood separates the two sections of urethra. This may feel like a boggy mass on rectal examination. Because of the rupture and hematoma, the prostate becomes high riding. A palpable prostate gland usually indicates a nonurethral injury. Absent sphincter tone would refer to a spinal cord injury. The presence of blood would probably correlate with GI bleeding or a colon injury.

Which statement describing urinary incontinence in the elderly is true? 1. Urinary incontinence is a normal part of aging. 2. Urinary incontinence isn't a disease. 3. Urinary incontinence in the elderly can't be treated. 4. Urinary incontinence is a disease.

2. Urinary incontinence isn't a disease. Urinary incontinence isn't a normal part of aging nor is it a disease. It may be caused by confusion, dehydration, fecal impaction, restricted mobility, or other causes. Certain medications, including diuretics, hypnotics, sedatives, anticholinergics, and antihypertensives, may trigger urinary incontinence. Most clients with urinary incontinence can be treated; some can be cured.

The nurse is reviewing the report of a client's routine urinalysis. Which value should the nurse consider abnormal? 1. Specific gravity of 1.03 2. Urine pH of 3.0 3. Absence of protein 4. Absence of glucose

2. Urine pH of 3.0 Normal urine pH is 4.5 to 8; therefore, a urine pH of 3.0 is abnormal. Urine specific gravity normally ranges from 1.002 to 1.035, making this client's value normal. Normally, urine contains no protein, glucose, ketones, bilirubin, bacteria, casts, or crystals. Red blood cells should measure 0 to 3 per high-power field; white blood cells, 0 to 4 per high-power field. Urine should be clear, with color ranging from pale yellow to deep amber.

An adult male is admitted to the emergency department with a strangulated inguinal hernia. Emergency surgery that involves reduction of the hernia, resection of a portion of the bowel, and repair of the abdominal wall is performed. What is most likely to be included in the immediate postoperative care? 1. Encouraging him to cough and deep breathe every 2 hours. 2. Using a scrotal support. 3. Frequently offering him oral fluids. 4. Inserting an indwelling urinary catheter.

2. Using a scrotal support. due to scrotal edema

Which conditions are functions of antidiuretic hormone (ADH)? 1. Sodium absorption and potassium excretion 2. Water reabsorption and urine concentration 3. Water reabsorption and urine dilution 4. Sodium reabsorption and potassium retention

2. Water reabsorption and urine concentration ADH stimulates the renal tubules to reabsorb water, thereby concentrating urine. Aldosterone is responsible for sodium reabsorption and potassium excretion by the kidneys.

A client with complaints of right, lower quadrant pain is admitted to the emergency department. Blood specimens are drawn and sent to the laboratory. Which laboratory finding should be reported to the physician immediately? 1. Hematocrit 42% 2. White blood cell (WBC) count 22.8/mm3 3. Serum potassium 4.2 mEq/L 4. Serum sodium 135 mEq/L

2. White blood cell (WBC) count 22.8/mm3 The nurse should report the elevated WBC count, which is evident in option 2. This finding, which is a sign of infection, indicates that the client's appendix might have ruptured. Hematocrit of 42%, serum potassium of 4.2 mEq/L, and serum sodium of 135 mEq/L are within normal limits. Alterations in these levels aren't indicative of appendicitis.

Because of difficulties with hemodialysis, peritoneal dialysis is initiated to treat a client's uremia. Which finding signals a significant problem during this procedure? 1. Blood glucose level of 200 mg/dl 2. White blood cell (WBC) count of 20,000/mm3 3. Potassium level of 3.5 mEq/L 4. Hematocrit (HCT) of 35%

2. White blood cell (WBC) count of 20,000/mm3 An increased WBC count indicates infection, probably resulting from peritonitis, which may have been caused by insertion of the peritoneal catheter into the peritoneal cavity. Peritonitis can cause the peritoneal membrane to lose its ability to filter solutes; therefore, peritoneal dialysis would no longer be a treatment option for this client. Hyperglycemia occurs during peritoneal dialysis because of the high glucose content of the dialysate; it's readily treatable with sliding-scale insulin. A potassium level of 3.5 mEq/L can be treated by adding potassium to the dialysate solution. An HCT of 35% is lower than normal. However, in this client, the value isn't abnormally low because of the daily blood samplings. A lower HCT is common in clients with chronic renal failure because of the lack of erythropoietin.

The nurse is inserting a urinary catheter into a client who is extremely anxious about the procedure. The nurse can facilitate the insertion by asking the client to: 1. initiate a stream of urine. 2. breathe deeply. 3. turn to the side. 4. hold the labia or shaft of the penis.

2. breathe deeply. When inserting a urinary catheter, facilitate insertion by asking the client to breathe deeply. Doing this will relax the urinary sphincter. Initiating a stream of urine isn't recommended during catheter insertion. Turning to the side or holding the labia or penis won't ease insertion, and doing so may contaminate the sterile field.

For a client who must undergo colon surgery, the physician orders preoperative cleansing enemas and neomycin sulfate (Mycifradin). The rationale for neomycin use in this client is to: 1. control postoperative nausea and vomiting. 2. decrease the intestinal bacteria count. 3. increase the intestinal bacteria count. 4. prevent the development of megacolon.

2. decrease the intestinal bacteria count. The antibiotic neomycin sulfate (Mycifradin) is prescribed to decrease the bacterial count and reduce the risk of fecal contamination during surgery. After surgery, the physician may prescribe an antiemetic — not an antibiotic — to control postoperative nausea and vomiting. Antibiotics decrease the intestinal bacteria count, not increase it. They have no relation to megacolon development. To prevent this complication, the client should avoid opioid analgesics, such as morphine (Duramorph), which can decrease intestinal motility and contribute to megacolon.

A client is scheduled to undergo surgical creation of an ileal conduit. The primary nurse educates the client about surgery and the postoperative period. The nurse informs the client that many members of the health care team (including a mental health practitioner) will see him. A mental health practitioner should be involved in the client's care to: 1. assess whether the client is a good candidate for surgery. 2. help the client cope with the anxiety associated with changes in body image. 3. assess suicidal risk postoperatively. 4. evaluate the client's need for mental health intervention.

2. help the client cope with the anxiety associated with changes in body image. Many clients who undergo surgery for creation of an ileal conduit experience anxiety associated with changes in body image. The mental health practitioner can help with client cope these feelings of anxiety. Mental health practitioners don't evaluate whether the client is a surgical candidate. None of the evidence suggests that urinary diversion surgery, such as creation of an ileal conduit, places the client at risk for suicide. Although evaluating the need for mental health intervention is always important, this client displays no behavioral changes that suggest intervention is necessary at this time.

The physician enters a computer order for the nurse to irrigate a client's nephrostomy tube every four hours to maintain patency. The nurse irrigates the tube using sterile technique. After irrigating the tube, the nurse decides that she can safely use the same irrigation set for her 8-hour shift if she covers the set with a paper, sterile drape. This action by the nurse is: 1. appropriate because the irrigation just checks for patency. 2. inappropriate because irrigation requires strict sterile technique. 3. appropriate because the irrigation set will only be used during an 8-hour period. 4. inappropriate because the sterile drape must be cloth, not paper.

2. inappropriate because irrigation requires strict sterile technique. Irritating a nephrostomy tube requires strict sterile technique; therefore, reusing the irrigation set (even if covered by a sterile drape) is inappropriate. Bacteria can proliferate inside the syringe and irrigation container. Although this procedure checks patency, it requires sterile technique to prevent the introduction of bacteria into the kidney. The material in which the sterile drape is made is irrelevant because a sterile drape doesn't deter bacterial growth in the irrigation equipment.

A client has undergone a colon resection. While turning him, wound dehiscence with evisceration occurs. The nurse's first response is to: 1. call the physician. 2. place saline-soaked sterile dressings on the wound. 3. take a blood pressure and pulse. 4. pull the dehiscence closed.

2. place saline-soaked sterile dressings on the wound. The nurse should first place saline-soaked sterile dressings on the open wound to prevent tissue drying and possible infection. Then the nurse should call the physician and take the client's vital signs. The dehiscence needs to be surgically closed, so the nurse should never try to close it.

A client with renal dysfunction of acute onset comes to the emergency department complaining of fatigue, oliguria, and coffee-colored urine. When obtaining the client's history to check for significant findings, the nurse should ask about: 1. chronic, excessive acetaminophen use. 2. recent streptococcal infection. 3. childhood asthma. 4. family history of pernicious anemia.

2. recent streptococcal infection. A skin or upper respiratory infection of streptococcal origin may lead to acute glomerulonephritis. Other infections that may be linked to renal dysfunction include infectious mononucleosis, mumps, measles, and cytomegalovirus. Chronic, excessive acetaminophen use isn't nephrotoxic, although it may be hepatotoxic. Childhood asthma and a family history of pernicious anemia aren't significant history findings for a client with renal dysfunction.

A client receiving total parental nutrition is prescribed a 24-hour urine test. When initiating a 24-hour urine specimen, the collection time should: 1. start with the first voiding. 2. start after a known voiding. 3. always be with the first morning urine. 4. always be the evening's last void as the last sample.

2. start after a known voiding. When initiating a 24-hour urine specimen, have the client void, then start the timing. The collection should start on an empty bladder. The exact time the test starts isn't important but it's commonly started in the morning.

A client with benign prostatic hypertrophy (BPH) asks a nurse what can be done to manage the symptoms. The correct response by the nurse would be: 1. "There is really nothing that can be done except surgical removal of the prostate." 2. "Increase the amount of the zinc in your diet." 3. "Herbal therapy such as saw palmetto may improve urinary flow." 4. "Vitamin E will reduce the size of the prostate."

3. "Herbal therapy such as saw palmetto may improve urinary flow." Dietary changes such as decreasing fats and increasing fruits may reduce symptoms. Reducing meat and dairy intake decreases hormone stimulation. Zinc has been shown to delay the development of BPH, but not reverse or manage the symptoms. Saw palmetto has been shown to improve urine flow bu blocking the ability of dihydrotestosterone (DHT) to stimulate prostate cell growth. Vitamin E has not been shown to reduce the size.

The nurse is doing preoperative teaching with the client who is about to undergo creation of a Kock pouch. The nurse interprets that the client has the best understanding of the nature of the surgery if the client makes which statement? 1. "I will be able to pass stool by the rectum eventually." 2. "The drainage from this type of ostomy will be formed." 3. "I will need to drain the pouch regularly with a catheter." 4. "I will need to wear a drainage bag for the rest of my life."

3. "I will need to drain the pouch regularly with a catheter." A Kock pouch is a continent ileostomy. As the ileostomy begins to function , the client drains it every 3 to 4 hours and then decreases the draining to about three times a day, or as needed when full. The client does not need to wear a drainage bag but should wear an absorbent dressing to absorb mucous drainage from the stoma. Ileostomy drainage is liquid. The client would be able to pass stool only from the rectum if an ileal-anal pouch or anastomosis were created. This type of operation is a two-stage procedure.

A client with acute diarrhea is prescribed paregoric, 5 ml by mouth up to four times daily, until acute diarrhea subsides. The client asks the nurse how soon the medication will start to work after the first dose is taken. How should the nurse respond? 1. "Within 5 minutes" 2. "Within 20 minutes" 3. "Within 1 hour" 4. "Within 2 to 4 hours"

3. "Within 1 hour" Paregoric starts to act within 1 hour after administration. Onset of action isn't as rapid as 5 or 20 minutes or as slow as 2 to 4 hours.

The registered nurse and nursing assistant are caring for a group of clients. Which client's care can safely be delegated to the nursing assistant? 1. A 35-year-old client who underwent surgery 12 hours ago and has a suprapubic catheter in place that is draining burgundy colored urine 2. A 63-year-old client with uncontrolled diabetes mellitus who underwent radical suprapubic prostatectomy 1 day ago and has an indwelling urinary catheter draining yellow urine with clots 3. A 45-year-old client diagnosed with renal calculi who must ambulate four times daily and drink plenty of fluids. 4. A 19-year-old client who requires neurological assessment every four hours after sustaining a spinal cord injury in a motor vehicle accident that left him with paraplegia

3. A 45-year-old client diagnosed with renal calculi who must ambulate four times daily and drink plenty of fluids. The care of the client in option 3 can safely be delegated to the nursing assistant. The client in option 1 had surgery 12 hours ago; therefore, the registered nurse should care for the client because the client requires close assessment. The client in option 2 also requires careful assessment by the registered nurse because the client's diabetes mellitus is uncontrolled. In addition, the registered nurse should care for the client in option 4 because the client requires neurological assessment, which isn't within the scope of practice for the nursing assistant.

After checking the client's chart for possible contraindications, the nurse is administering meperidine (Demerol), 50 mg I.M., to a client with pain after an appendectomy. Which type of drug therapy would contraindicate the use of meperidine? 1. An antibiotic 2. An antiemetic 3. A monoamine oxidase (MAO) inhibitor 4. A loop diuretic

3. A monoamine oxidase (MAO) inhibitor MAO inhibitors increase the effects of meperidine and can cause rigidity, hypotension, and excitation. The client shouldn't receive meperidine within 14 days after administration of an MAO inhibitor. Antibiotics, antiemetics, and loop diuretics don't cause significant drug interactions when administered concurrently with meperidine.

The nurse correctly identifies a urine sample with a pH of 4.3 as being which type of solution? 1. Neutral 2. Alkaline 3. Acidic 4. Basic

3. Acidic Normal urine pH is 4.5 to 8.0; a value of 4.3 reveals acidic urine pH. A pH above 7.0 is considered an alkaline or basic solution. A pH of 7.0 is considered neutral.

A client has just had a hemorrhoidectomy. What nursing intervention is appropriate for this client? 1. Instruct the client to limit fluid intake to avoid urinary retention. 2. Instruct the client to eat low-fiber foods to decrease the bulk of the stool. 3. Apply and maintain ice packs over the dressing until the packing is removed. 4. Help the client to a Fowler's position to place pressure on the rectal area and decrease bleeding.

3. Apply and maintain ice packs over the dressing until the packing is removed. Nursing interventions after a hemorrhoidectomy are aimed at management of pain and avoidance of bleeding. An ice pack will increase comfort and decrease bleeding. Options 1, 2, and 4 are incorrect interventions.

The home care nurse is making a visit with a client who had a double barrel colostomy created after bowel surgery. While the nurse is changing the client's appliance there is a knock on the door. The nurse answers the door. The client's next-door neighbor wants to visit with the client. Which intervention by the nurse is most appropriate? 1. Allow the neighbor to enter. 2. Have the neighbor wait in the next room until the appliance is applied. 3. Ask the neighbor to come back in 20 minutes. 4. Suggest that the neighbor come in and learn how to apply the appliance in case the client needs help.

3. Ask the neighbor to come back in 20 minutes. The home care nurse should ask the neighbor to come back in 20 minutes when the visit is over. Client privacy is a priority even in the home care setting. Options 1 and 2 violate client privacy and confidentiality. Option 4 is inappropriate because the client didn't request help from the neighbor.

A client who underwent abdominal surgery who has a nasogastric (NG) tube in place begins to complain of abdominal pain that he describes as "feeling full and uncomfortable." Which assessment should the nurse perform first? 1. Measure abdominal girth. 2. Auscultate bowel sounds. 3. Assess patency of the NG tube. 4. Assess vital signs.

3. Assess patency of the NG tube. When an NG tube is no longer patent, stomach contents collect in the stomach giving the client a sensation of fullness. The nurse should begin by assessing patency of the NG tube. The nurse can measure abdominal girth, auscultate bowels, and assess vital signs, but she should check NG tube patency first to help relieve the client's discomfort.

After undergoing transurethral resection of the prostate to treat benign prostatic hyperplasia, a client returns to the room with continuous bladder irrigation. On the first day after surgery, the client reports bladder pain. What should the nurse do first? 1. Increase the I.V. flow rate. 2. Notify the physician immediately. 3. Assess the irrigation catheter for patency and drainage. 4. Administer morphine sulfate, 2 mg I.V., as prescribed.

3. Assess the irrigation catheter for patency and drainage. Although postoperative pain is expected, the nurse should make sure that other factors, such as an obstructed irrigation catheter, aren't the cause of the pain. After assessing catheter patency, the nurse should administer an analgesic, such as morphine sulfate, as prescribed. Increasing the I.V. flow rate may worsen the pain. Notifying the physician isn't necessary unless the pain is severe or unrelieved by the prescribed medication.

A client with acute pyelonephritis receives a prescription for co-trimoxazole (Septra) P.O. twice daily for 10 days. Which finding best demonstrates that the client has followed the prescribed regimen? 1. Urine output increases to 2,000 ml/day. 2. Flank and abdominal discomfort decrease. 3. Bacteria are absent on urine culture. 4. The red blood cell (RBC) count is normal.

3. Bacteria are absent on urine culture.Co-trimoxazole is a sulfonamide antibiotic used to treat urinary tract infections. Therefore, absence of bacteria on urine culture indicates that the drug has achieved its desired effect. Although flank pain may decrease as the infection resolves, this isn't a reliable indicator of the drug's effectiveness. Co-trimoxazole doesn't affect urine output or the RBC count.

A client develops decreased renal function and requires a change in antibiotic dosage. On which factor would the physician base the dosage change? 1. GI absorption rate 2. Therapeutic index 3. Creatinine clearance 4. Liver function studies

3. Creatinine clearance The physician orders tests for creatinine clearance to gauge the kidney's glomerular filtration rate; this is important because most drugs are excreted at least partially by the kidneys. The GI absorption rate, therapeutic index, and liver function studies don't help determine dosage change in a client with decreased renal function.

What should be the first nursing action if an NG tube is not draining? 1. Irrigate the tube. 2. Reposition the client. 3. Determine tube placement. 4. Remove the tube and reinsert.

3. Determine tube placement. Placement must be confirmed before irrigating. The lack of drainage may mean the tube is no longer in the correct location. Placement must be determined before removal.

A client complains of not having had a bowel movement since being admitted 2 days previously for multiple fractures of both lower legs. The client is on bedrest and skeletal traction. Which intervention would be the most appropriate nursing action? 1. Administer an enema. 2. Put the client on the bedpan every 2 hours. 3. Ensure maximum fluid intake (3,000mL/day) 4. Perform range-of-motion exercises to all extremities.

3. Ensure maximum fluid intake (3,000mL/day) Enema may not be necessary and requires a doctor's order. The bedpan requires a great deal of exertion when the client is not expressing the urge to defecate. The best early intervention is to increase fluid intake b/ constipation is common when activity is decreased or usual routines have been interrupted. It would be impossible to exercise extremities that have unhealed fractures.

Which statement by the nurse correctly describes a double-barrel colostomy? 1. It is the least common type of colostomy, and it discharges liquid or unformed stool. 2. A single loop of the transverse colon is exteriorized and supported by a glass rod. There are two openings, a proximal loop and distal loop. 3. It has two stomas. A proximal loop discharges feces, and a distal loop discharges mucus. 4. It is most often permanent and is done to treat disorders of the sigmoid colon.

3. It has two stomas. A proximal loop discharges feces, and a distal loop discharges mucus. Number 1 is an example of an ascending colostomy. Number 2 is a transverse loop colostomy. Number 4 is a descending colostomy.

After undergoing retropubic prostatectomy, a client returns to his room. The client is on nothing-by-mouth status and has an I.V. infusing in his right forearm at a rate of 100 ml/hour. The client also has an indwelling urinary catheter that is draining light pink urine. While assessing the client, the nurse notes that his urine output is red and has dropped to 15 ml and 10 ml for the last two consecutive hours. How can the nurse best explain this drop in urine output? 1. It's a normal finding caused by blood loss during surgery. 2. It's a normal finding associated with the client's nothing-by-mouth status. 3. It's an abnormal finding that requires further assessment. 4. It's an abnormal finding that will correct itself when the client ambulates.

3. It's an abnormal finding that requires further assessment. The drop in urine output to less than 30 ml/hour is abnormal and requires further assessment. The reduction in urine output may be caused by an obstruction in the urinary catheter tubing or deficient fluid volume from blood loss. The client's nothing-by-mouth status isn't the cause of the low urine output because the client is receiving I.V. fluid to compensate for the lack of oral intake. Ambulation promotes urination; however, the client should produce at least 30 ml of urine/hour.

The nurse is planning a group teaching session on the topic of urinary tract infection (UTI) prevention. Which point would the nurse want to include? 1. Limit fluid intake to reduce the need to urinate. 2. Take medication prescribed for a UTI until the symptoms subside. 3. Notify the physician if urinary urgency, burning, frequency, or difficulty occurs. 4. Wear only nylon underwear to reduce the chance of irritation.

3. Notify the physician if urinary urgency, burning, frequency, or difficulty occurs. Urgency, burning, frequency, and difficulty urinating are all common symptoms of a UTI. The client should notify his physician so that a microscopic urinalysis can be done and appropriate treatment can be initiated. The client should be instructed to drink 2 to 3 L of fluid per day to dilute the urine and reduce irritation of the bladder mucosa. The full amount of antibiotics prescribed for UTIs must be taken despite the fact that the symptoms may have subsided. This will help to prevent recurrences of UTI. Women are told to avoid scented toilet tissue and bubble baths and to wear cotton underwear, not nylon, to reduce the chance of irritation.

Which steps should the nurse follow to insert a straight urinary catheter? 1. Create a sterile field, drape the client, clean the meatus, and insert the catheter only 6". 2. Put on gloves, prepare equipment, create a sterile field, expose the urinary meatus, and insert the catheter 6". 3. Prepare the client and equipment, create a sterile field, put on gloves, clean the urinary meatus, and insert the catheter until urine flows. 4. Prepare the client and equipment, create a sterile field, test the catheter balloon, clean the meatus, and insert the catheter until urine flows.

3. Prepare the client and equipment, create a sterile field, put on gloves, clean the urinary meatus, and insert the catheter until urine flows. Option 3 describes all the vital steps for inserting a straight catheter. Option 1 is incorrect because the nurse must prepare the client and equipment before creating a sterile field. Option 2 is incorrect because the nurse put on gloves before creating a sterile field and performing the other tasks. Option 4 describes the procedure for inserting a retention catheter, rather than a straight catheter.

A client is scheduled for a renal arteriogram. When the nurse checks the chart for allergies to shellfish or iodine, the nurse finds no allergies recorded. The client is unable to provide the information. During the procedure, the nurse should be alert for which finding that may indicate an allergic reaction to the dye used during the arteriogram. 1. Increased alertness 2. Hypoventilation 3. Pruritus 4. Unusually smooth skin

3. Pruritus The nurse should be alert for urticaria and pruritus, which may indicate a mild anaphylactic reaction to the arteriogram dye. Decreased (not increased) alertness may occur as well as dyspnea (not hypoventilation). Unusually smooth skin isn't a sign of anaphylaxis.

Which symptom is a warning sign of colon cancer? 1. Hoarseness 2. Indigestion 3. Rectal bleeding 4. Sore that doesn't heal

3. Rectal bleeding Rectal bleeding of dark to bright red blood is a warning sign of colon cancer. Hoarseness, indigestion, and a sore that doesn't heal are potential warning signs of other types of cancer.

The purpose of giving neomycin before ileostomy surgery is to: 1. Decrease the incidence of postoperative atelectasis due to decreased depth of respirations. 2. Increase the effectiveness of the body's immunologic response following surgical trauma. 3. Reduce the incidence of wound infections by decreasing the number of intestinal organisms. 4. Prevent postoperative bladder atony due to catheterization.

3. Reduce the incidence of wound infections by decreasing the number of intestinal organisms. Neomycin is not effective in reducing postop atelectasis. The ability of the body to ward off infection is a result of the decrease in intestinal organisms. Neomycin is administered postoperatively because it is a poorly absorbed antibiotic and therefore is effective in reducing the number of intestinal organisms that may cause infection of the suture line. Bladder atony is generally due to decreased parasympathetic outflow and bladder tone secondary to anesthesia.

A 37-year-old man with ulcerative colitis has a new ileostomy. To promote a positive self-image for the client, the nurse will: 1. Teach the client to inspect the stoma daily. 2. Invite the client's wife and two sons to observe while the client changes the ostomy appliance. 3. Support the client in changing the ostomy appliance, but realize he may not like it. 4. Acknowledge the presence of odor during ostomy appliance changes by holding her breath.

3. Support the client in changing the ostomy appliance, but realize he may not like it. Teaching the client to inspect the stoma daily does not directly address the client's self-image. Providing privacy is appropriate unless the client gives permission for the family to observe and they will be participating in the care of the stoma. Answer 3 is correct because this behavior conveys to the client that the nurse understands and accepts some of his feelings about the new ostomy. If the nurse expresses distaste, verbally or nonverbally, this reinforces the distastefulness of the ostomy to the client.

The physician calls the nurse for an update on his client who underwent abdominal surgery five hours ago. The physician asks the nurse for the total amount of drainage collected in the Hemovac since surgery. The nurse reports that according to documentation, no drainage has been recorded. When the nurse finishes on the telephone, she goes to assess the client. Which assessment finding explains the absence of drainage? 1. The client has been lying on his side for two hours with the drain positioned upward. 2. The client has a nasogastric (NG) tube in place that drained 400 ml. 3. The Hemovac drain isn't compressed; instead it's fully expanded. 4. There is a moderate amount of dry drainage on the outside of the dressing.

3. The Hemovac drain isn't compressed; instead it's fully expanded. The Hemovac must be compressed to establish suction. If the Hemovac is allowed to fully expand suction is no longer present causing the drain to malfunction. The client who requires major abdominal surgery typically produces abdominal drainage despite the client's position. An NG tube drains stomach contents, not incisional contents. Therefore, the NG tube drainage of 400 ml is normal in this client and is not related to the absence of Hemovac drainage. Dry drainage on the dressing indicates leakage from the incision; it isn't related to the Hemovac drainage.

A client with suspected renal insufficiency is scheduled for a comprehensive diagnostic workup. After the nurse explains the diagnostic tests, the client asks which part of the kidney "does the work." Which answer is correct? 1. The glomerulus 2. Bowman's capsule 3. The nephron 4. The tubular system

3. The nephron The nephron is the functioning unit of the kidney. The glomerulus, Bowman's capsule, and tubular system are components of the nephron.

Which statement best describes the therapeutic action of loop diuretics? 1. They block reabsorption of potassium on the collecting tubule. 2. They promote sodium secretion into the distal tubule. 3. They block sodium reabsorption in the ascending loop and dilate renal vessels. 4. They promote potassium secretion into the distal tubule and constrict renal vessels.

3. They block sodium reabsorption in the ascending loop and dilate renal vessels. Loop diuretics block sodium reabsorption in the ascending loop of Henle, which promotes water diuresis. They also dilate renal vessels. Loop diuretics block potassium reabsorption, but this isn't a therapeutic effect. Thiazide diuretics promote sodium secretion into the distal tubule.

The client arrives at the emergency department with complaints of low abdominal pain and hematuria. The client is afebrile. The nurse next assesses the client to determine a history of: 1. Pyelonephritis 2. Glomerulonephritis 3. Trauma to the bladder or abdomen 4. Renal cancer in the client's family

3. Trauma to the bladder or abdomen Bladder trauma or injury should be considered or suspected in the client with low abdominal pain and hematuria. Glomerulonephritis and pyelonephritis would be accompanied by fever and are thus not applicable to the client described in this question. Renal cancer would not cause pain that is felt in the low abdomen; rather pain would be in the flank area.

A client in the short-procedure unit is recovering from renal angiography in which a femoral puncture site was used. When providing postprocedure care, the nurse should: 1. keep the client's knee on the affected side bent for 6 hours. 2. apply pressure to the puncture site for 30 minutes. 3. check the client's pedal pulses frequently. 4. remove the dressing on the puncture site after vital signs stabilize.

3. check the client's pedal pulses frequently. After renal angiography involving a femoral puncture site, the nurse should check the client's pedal pulses frequently to detect reduced circulation to the feet caused by vascular injury. The nurse also should monitor vital signs for evidence of internal hemorrhage and should observe the puncture site frequently for fresh bleeding. The client should be kept on bed rest for several hours so the puncture site can seal completely. Keeping the client's knee bent is unnecessary. By the time the client returns to the short-procedure unit, manual pressure over the puncture site is no longer needed because a pressure dressing is in place. The nurse shouldn't remove this dressing for several hours — and only if instructed to do so.

The physician orders a stool culture to help diagnose a client with prolonged diarrhea. The nurse who obtains the stool specimen should: 1. take the specimen to the laboratory immediately. 2. apply a solution to the stool specimen. 3. collect the specimen in a sterile container. 4. store the specimen on ice.

3. collect the specimen in a sterile container. The nurse should collect the stool specimen using sterile technique and a sterile stool container. The stool may be collected for 3 consecutive days; no follow-up care is needed. Although a stool culture should be taken to the laboratory as soon as possible, it need not be delivered immediately (unlike stool being examined for ova and parasites). Applying a solution to a stool specimen would contaminate it; this procedure is done when testing stool for occult blood, not organisms. The nurse shouldn't store a stool culture on ice because the abrupt temperature change could kill the organisms.

When planning care for a client with a small-bowel obstruction, the nurse should consider the primary goal to be: 1. reporting pain relief. 2. maintaining body weight. 3. maintaining fluid balance. 4. reestablishing a normal bowel pattern.

3. maintaining fluid balance. Because a client with a small-bowel obstruction can't tolerate oral intake, fluid volume deficit may occur and can be life-threatening. Therefore, maintaining fluid balance is the primary goal. The other options are secondary; pain relief and maintaining body weight don't reflect life-threatening conditions, and the client's normal bowel pattern can be reestablished after fluid volume is stabilized.

In a client with enteritis and frequent diarrhea, the nurse should anticipate an acid-base imbalance of: 1. respiratory acidosis. 2. respiratory alkalosis. 3. metabolic acidosis. 4. metabolic alkalosis.

3. metabolic acidosis. Diarrhea causes a bicarbonate deficit. With loss of the relative alkalinity of the lower GI tract, the relative acidity of the upper GI tract predominates leading to metabolic acidosis. Loss of acid, which occurs with severe vomiting, may lead to metabolic alkalosis. Diarrhea doesn't lead to respiratory acid-base imbalances, such as respiratory acidosis and respiratory alkalosis.

The nurse is providing postprocedure care for a client who underwent percutaneous lithotripsy. In this procedure, an ultrasonic probe inserted through a nephrostomy tube into the renal pelvis generates ultra-high-frequency sound waves to shatter renal calculi. The nurse should instruct the client to: 1. limit oral fluid intake for 1 to 2 weeks. 2. report the presence of fine, sandlike particles through the nephrostomy tube. 3. notify the physician about cloudy or foul-smelling urine. 4. report bright pink urine within 24 hours after the procedure.

3. notify the physician about cloudy or foul-smelling urine. The client should report the presence of foul-smelling or cloudy urine. Unless contraindicated, the client should be instructed to drink large quantities of fluid each day to flush the kidneys. Sandlike debris is normal because of residual stone products. Hematuria is common after lithotripsy.

A 68-year-old male is being admitted to the hospital with abdominal pain, anemia, and bloody stools. He complains of feeling weak and dizzy. He has rectal pressure and needs to urinate and move his bowels. The nurse should help him: 1. to the bathroom. 2. to the bedside commode. 3. onto the bedpan. 4. to a standing position so he can urinate.

3. onto the bedpan. A client who's dizzy and anemic is at risk for injury because of his weakened state. Assisting him with the bedpan would best meet his needs at this time without risking his safety. The client may fall if walking to the bathroom, left alone to urinate, or trying to stand up.

A female client reports to the nurse that she experiences a loss of urine when she jogs. The nurse's assessment reveals no nocturia, burning, discomfort when voiding, or urine leakage before reaching the bathroom. The nurse explains to the client that this type of problem is called: 1. functional incontinence. 2. reflex incontinence. 3. stress incontinence. 4. total incontinence.

3. stress incontinence. Stress incontinence is a small loss of urine with activities that increase intra-abdominal pressure, such as running, laughing, sneezing, jumping, coughing, and bending. These symptoms occur only in the daytime. Functional incontinence is the inability of a usually continent client to reach the toilet in time to avoid unintentional loss of urine. Reflex incontinence is an involuntary loss of urine at predictable intervals when a specific bladder volume is reached. Total incontinence occurs when a client experiences a continuous and unpredictable loss of urine.

During a client-teaching session, which instruction should the nurse give to a client receiving kaolin and pectin (Kaopectate) for treatment of diarrhea? 1. "Take the medication after every other loose bowel movement, up to five doses per day." 2. "Avoid taking this medication for more than 4 days." 3. "Consult the physician if you have more than six bowel movements in 1 day." 4. "Drink 8 to 13 8-oz glasses (2 to 3 L) of fluid daily."

4. "Drink 8 to 13 8-oz glasses (2 to 3 L) of fluid daily." The nurse should tell the client to drink 8 to 13 8-oz glasses of fluid daily to replace fluids lost through diarrhea. Kaolin and pectin mixtures should be taken after each loose bowel movement for up to eight doses daily. The client should avoid self-medication for longer than 48 hours. The client should consult a physician if diarrhea persists longer than 48 hours despite treatment.

The spouse of a client who had a transurethral resection of the prostate (TURP) 24 hours ago is upset because the irrigation seems to be increasing the client's pain. Which is the best response by a nurse? 1. "I will stop the bladder irrigation and the pain should subside." 2. "I will add a local anesthetic to the irrigant to treat the pain." 3. "Perhaps you should go home and get some rest." 4. "The rate of irrigant should be slowed when the drainage is pale pink."

4. "The rate of irrigant should be slowed when the drainage is pale pink." The nurse cannot stop the irrigation without an order. The nurse does not have the authority to add anesthetic. The purpose of the irrigant is to prevent clots. A pale pink would indicate that bleeding is diminishing.

The nurse is assessing the stoma of a client following a ureterostomy. Which of the following should the nurse expect to note? 1. A dry stoma 2. A pale stoma 3. A dark-colored stoma 4. A red and moist stoma

4. A red and moist stoma Following ureterostomy, the stoma should be red and moist. A pale stoma may indicate an inadequate amount of vascular supply. A dry stoma may indicate a body fluid deficit. Any sign of darkness or duskiness in the stoma may indicate a loss of vascular supply and must be reported immediately or necrosis can occur.

The nurse caring for a client with small-bowel obstruction would plan to implement which nursing intervention first? 1. Administering pain medication 2. Obtaining a blood sample for laboratory studies 3. Preparing to insert a nasogastric (NG) tube 4. Administering I.V. fluids

4. Administering I.V. fluids I.V. infusions containing normal saline solution and potassium should be given first to maintain fluid and electrolyte balance. For the client's comfort and to assist in bowel decompression, the nurse should prepare to insert an NG tube next. A blood sample is then obtained for laboratory studies to aid in the diagnosis of bowel obstruction and guide treatment. Blood studies usually include a complete blood count, serum electrolyte levels, and blood urea nitrogen level. Pain medication often is withheld until obstruction is diagnosed because analgesics can decrease intestinal motility.

A client is frustrated and embarrassed by urinary incontinence. Which measure should the nurse include in a bladder retraining program? 1. Establishing a predetermined fluid intake pattern for the client 2. Encouraging the client to increase the time between voidings 3. Restricting fluid intake to reduce the need to void 4. Assessing present elimination patterns

4. Assessing present elimination patterns The guidelines for initiating bladder retraining include assessing the client's intake patterns, voiding patterns, and reasons for each accidental voiding. Lowering the client's fluid intake won't reduce or prevent incontinence. The client should actually be encouraged to drink 1.5 to 2 L of water per day. A voiding schedule should be established after assessment.

The client is admitted to the hospital with a diagnosis of benign prostatic hyperplasia, and a transurethral resection of the prostate is performed. Four hours after surgery, the nurse takes the client's vital signs and empties the urinary drainage bag. Which of the following assessment findings would indicate the need to notify the physician? 1. Red bloody urine 2. Pain related to bladder spasms 3. Urinary output of 200 mL higher than intake 4. Blood pressure, 100/ 50 mm Hg; pulse, 130 beats/ min

4. Blood pressure, 100/ 50 mm Hg; pulse, 130 beats/ min Frank bleeding (arterial or venous) may occur during the first day after surgery. Some hematuria is usual for several days after surgery. A urinary output of 200 mL more than intake is adequate. Bladder spasms are expected to occur following surgery. A rapid pulse with a low blood pressure is a potential sign of excessive blood loss. The physician should be notified.

The physician prescribes a single dose of trimethoprim/sulfamethoxazole (Bactrim) by mouth for a client diagnosed with an uncomplicated urinary tract infection (UTI). The pharmacy sends three unit-dose tablets. The nurse verifies the physician's order. What should the nurse do next? 1. Administer the three tablets as the single dose. 2. Call the physician to verify the order. 3. Give one tablet, three times per day. 4. Call the hospital pharmacist and question the medication supplied.

4. Call the hospital pharmacist and question the medication supplied. The nurse should call the hospital pharmacy and question the medication supplied. The hospital pharmacist should be able to tell the nurse whether three tablets are necessary for the single dose or whether a dispensing error occurred. It isn't clear whether the three tablets are the single dose because they were packaged as a unit-dose. The physician order was clearly written, so clarifying the order with the physician isn't necessary. Administering the tablets without clarification might cause a medication error.

The client is scheduled for urinary diversion surgery to treat bladder cancer. Before surgery, the health care team consisting of a nurse, dietician, social worker, enterostomal therapist, surgeon, client educator, and mental health worker meet with the client. After the meeting, the client states, "My life won't ever be the same. What am I going to do?" Which health team member should the nurse consult to help with the client's concerns? 1. Social worker 2. Surgeon 3. Dietician 4. Client educator

4. Client educator The nurse should consult the client educator to help the client with his fears and concerns. Providing the client with information can greatly allay the client's fears. The social worker can provide the client with services he may need after discharge. The dietician can help with dietary concerns but can't provide help with direct concerns about the surgery.

During clindamycin (Cleocin) therapy, the nurse monitors a client for pseudomembranous colitis. This serious adverse reaction to clindamycin results from superinfection with which organism? 1. Staphylococcus aureus 2. Bacteroides fragilis 3. Escherichia coli 4. Clostridium difficile

4. Clostridium difficile Pseudomembranous colitis may result from a superinfection with C. difficile during clindamycin therapy. Clindamycin-induced pseudomembranous colitis isn't caused by S. aureus, B. fragilis, or E. coli.

A 28-year-old client is admitted with inflammatory bowel syndrome (Crohn's disease). Which therapies should the nurse expect to be part of the care plan? 1. Lactulose therapy 2. High-fiber diet 3. High-protein milkshakes 4. Corticosteroid therapy 5. Antidiarrheal medications

4. Corticosteroid therapy 5. Antidiarrheal medications Corticosteroids, such as prednisone, reduce the signs and symptoms of diarrhea, pain, and bleeding by decreasing inflammation. Antidiarrheals, such as diphenoxylate (Lomotil), combat diarrhea by decreasing peristalsis. Lactulose is used to treat chronic constipation and would aggravate the symptoms. A high-fiber diet and milk and milk products are contraindicated in clients with Crohn's disease because they may promote diarrhea.

The nurse is taking the history of a client who has had benign prostatic hyperplasia in the past. To determine whether the client currently is experiencing difficulty, the nurse asks the client about the presence of which early symptom? 1. Nocturia 2. Urinary retention 3. Urge incontinence 4. Decreased force in the stream of urine

4. Decreased force in the stream of urine Decreased force in the stream of urine is an early sign of benign prostatic hyperplasia. The stream later becomes weak and dribbling. The client then may develop hematuria, frequency, urgency, urge incontinence, and nocturia. If untreated, complete obstruction and urinary retention can occur.

The male client has a tentative diagnosis of urethritis. The nurse assesses the client for which of the following manifestations of the disorder? 1. Hematuria and pyuria 2. Dysuria and proteinuria 3. Hematuria and urgency 4. Dysuria and penile discharge

4. Dysuria and penile discharge Urethritis in the male client often results from chlamydial infection and is characterized by dysuria, which is accompanied by a clear to mucopurulent discharge. Because this disorder often coexists with gonorrhea, diagnostic tests are done for both and include culture and rapid assays.

An elderly client continues to have fecal incontinence with 6 to 7 small brown liquid stools each day. The client eats a soft diet and does not receive any stool softeners or laxatives. The client's primary form of activity is sitting in the wheelchair for 2 hours twice a day. What is the correct explanation for the frequent diarrhea stools? 1. Inadequate roughage in the diet. 2. Inactivity from sedentary lifestyle. 3. Gastrointestinal virus. 4. Fecal impaction.

4. Fecal impaction. Inadequate roughage would not cause diarrhea. Fecal impaction is likely causing diarrhea. Fiber can be added to a soft diet to prevent constipation. Inactivity contributes to the potential for constipation and fecal impaction, but the impaction is causing the liquid stools. Pressure on the colonic mucosa causes seepage of liquid stool around the area of impaction.

The client has just had surgery to create an ileostomy. The nurse assesses the client in the immediate postoperative period for which most frequent complication of this type of surgery? 1. Folate deficiency 2. Malabsorption of fat 3. Intestinal obstruction 4. Fluid and electrolyte imbalance

4. Fluid and electrolyte imbalance A frequent complication that occurs following ileostomy is fluid and electrolyte imbalance. The client requires constant monitoring of intake and output to prevent this from occurring. Losses require replacement by intravenous infusion until the client can tolerate a diet orally. Intestinal obstruction is a less frequent complication. Fat malabsorption and folate deficiency are complications that could occur later in the postoperative period.

An adult is admitted with probable large bowel obstruction. The nurse auscultates the abdomen. Which finding, if present, is consistent with an intestinal obstruction? 1. Hyperactive bowel sounds in all 4 quadrants. 2. Hypoactive bowel sounds in all 4 quadrants. 3. Coffee grounds emesis and tarry stools. 4. High pitched bowel sounds in the right quadrants; hypoactive bowel sounds in the left quadrants.

4. High pitched bowel sounds in the right quadrants; hypoactive bowel sounds in the left quadrants.

A client is scheduled to undergo an exploratory laparoscopy. The registered nurse asks the licensed practical nurse (LPN) to prepare the client for surgery. The registered nurse must confirm that the LPN has specialized training before delegating which task? 1. Weighing the client 2. Teaching the client coughing and deep breathing exercises 3. Teaching the client how to collect a urine specimen 4. Initiating intravenous therapy, as ordered

4. Initiating intravenous therapy, as ordered The registered nurse must confirm that the LPN has specialized I.V. training before asking her to begin I.V. therapy for this client. Initiating I.V. therapy is beyond the usual scope of practice for an LPN. Options 1, 2, and 3 are within the scope of LPN practice and don't require additional training.

What should the nurse include in health teaching for a client who has an ileostomy? 1. Eat high-fiber foods. 2. Take laxatives for any blockage. 3. Eat raw fruits and vegetables. 4. Maintain adequate fluid intake.

4. Maintain adequate fluid intake. High-fiber foods should be avoided. Laxatives are not needed; drainage is liquid. Increased peristalsis may contribute to FVD and discomfort. Certain raw fruits and vegetables have high cellulose content and should be avoided because they could cause blockage. Due to lack of reabsorption of fluid from the large intestine, the client is at risk for FVD.

A client with a history of chronic cystitis comes to the outpatient clinic with signs and symptoms of this disorder. To prevent cystitis from recurring, the nurse recommends maintaining an acid-ash diet to acidify the urine, thereby decreasing the rate of bacterial multiplication. On an acid-ash diet, the client must restrict which beverage? 1. Cranberry juice 2. Coffee 3. Prune juice 4. Milk

4. Milk A client on an acid-ash diet must avoid milk and milk products because these make the urine more alkaline, encouraging bacterial growth. Other foods to avoid on this diet include all vegetables except corn and lentils; all fruits except cranberries, plums, and prunes; and any food containing large amounts of potassium, sodium, calcium, or magnesium. Cranberry and prune juice are encouraged because they acidify the urine. Coffee and tea are considered neutral because they don't alter the urine pH.

A client with nausea, vomiting, and abdominal cramps and distention is admitted to the health care facility. Which test result is most significant? 1. Blood urea nitrogen (BUN) level of 29 mg/dl 2. Serum sodium level of 132 mEq/L 3. Urine specific gravity of 1.025 4. Serum potassium level of 3 mEq/L

4. Serum potassium level of 3 mEq/L A serum potassium level of 3 mEq/L is below normal, indicating hypokalemia. Because hypokalemia may cause cardiac arrhythmias and asystole, it's the most significant finding. In a client with a potential fluid volume imbalance, such as from vomiting, the other options are expected and none is as life-threatening as hypokalemia. A BUN level of 29 mg/dl indicates slight dehydration, probably caused by vomiting. A serum sodium level of 132 mEq/L is slightly below normal but not life-threatening. A urine specific gravity of 1.025 is normal.

A client is admitted with nausea, vomiting, and diarrhea. His blood pressure on admission is 74/30 mm Hg. The client is oliguric and his blood urea nitrogen (BUN) and creatinine levels are elevated. The physician will most likely write an order for which treatment? 1. Force oral fluids. 2. Administer furosemide (Lasix) 20 mg I.V. 3. Start hemodialysis after a temporary access is obtained. 4. Start I.V. fluids with a normal saline solution bolus followed by a maintenance dose.

4. Start I.V. fluids with a normal saline solution bolus followed by a maintenance dose. The client is in prerenal failure caused by hypovolemia. I.V. fluids should be given with a bolus of normal saline solution followed by maintenance I.V. therapy. This treatment should rehydrate the client, causing his blood pressure to rise, his urine output to increase, and the BUN and creatinine levels to normalize. The client wouldn't be able to tolerate oral fluids because of the nausea, vomiting, and diarrhea. The client isn't fluid-overloaded so his urine output won't increase with furosemide, which would actually worsen the client's condition. The client doesn't require dialysis because the oliguria and elevated BUN and creatinine levels are caused by dehydration.

A client with heart failure admitted to an acute care facility and is found to have a cystocele. When planning care for this client, the nurse is most likely to formulate which nursing diagnosis? 1. Total urinary incontinence 2. Functional urinary incontinence 3. Reflex urinary incontinence 4. Stress urinary incontinence

4. Stress urinary incontinence Stress urinary incontinence is a urinary problem associated with cystocele — herniation of the bladder into the birth canal. Other problems associated with this disorder include urinary frequency, urinary urgency, urinary tract infection (UTI), and difficulty emptying the bladder. Total incontinence, functional incontinence, and reflex incontinence usually result from neurovascular dysfunction, not cystocele.

Which outcome indicates effective client teaching to prevent constipation? 1. The client verbalizes consumption of low-fiber foods. 2. The client maintains a sedentary lifestyle. 3. The client limits water intake to three glasses per day. 4. The client reports engaging in a regular exercise regimen.

4. The client reports engaging in a regular exercise regimen. A regular exercise regimen promotes peristalsis and contributes to regular bowel elimination patterns. A low-fiber diet, a sedentary lifestyle, and limited water intake would predispose the client to constipation.

A client is scheduled to undergo a left hemicolectomy for colorectal cancer. The physician prescribes phenobarbital (Luminal), 100 mg I.M. 60 minutes before surgery, for sedation. Which statement accurately describes administration of phenobarbital? 1. The onset of action for I.M. administration is 30 to 60 minutes. 2. This drug can be mixed and given with other medications. 3. This drug should be used within 24 hours after opening. 4. This drug should be injected into a large muscle mass.

4. This drug should be injected into a large muscle mass. Phenobarbital should be injected into a large muscle mass. The onset of action by the I.M. route is 10 to 30 minutes. Barbiturates are involved in many drug interactions, so the drug shouldn't be mixed or given with other medications. The drug solution should be used within 30 minutes after opening to minimize deterioration.

The client underwent a transurethral resection of the prostate gland 24 hours ago and is on continuous bladder irrigation. Which nursing intervention is appropriate? 1. Tell the client to try to urinate around the catheter to remove blood clots. 2. Restrict fluids to prevent the client's bladder from becoming distended. 3. Prepare to remove the catheter. 4. Use aseptic technique when irrigating the catheter.

4. Use aseptic technique when irrigating the catheter. If the catheter is blocked by blood clots, it may be irrigated according to physician's orders or facility protocol. The nurse should use sterile technique to reduce the risk of infection. Urinating around the catheter can cause painful bladder spasms. Encourage the client to drink fluids to dilute the urine and maintain urine output. The catheter remains in place for 2 to 4 days after surgery and is only removed with a physician's order.

Following an earthquake, a client who was rescued from a collapsed building is seen in the emergency department. He has blunt trauma to the thorax and abdomen. The nursing observation that most suggests the client is bleeding is: 1. a prolonged partial thromboplastin time (PTT). 2. a recent history of warfarin (Coumadin) usage. 3. diminished breath sounds. 4. orthostatic hypotension.

4. orthostatic hypotension. Bleeding is a volume-loss problem, which causes a drop in blood pressure. As the bleeding persists and the body's ability to compensate declines, orthostatic hypotension becomes evident. A prolonged PTT and a history of warfarin usage are causes of bleeding but aren't evidence of bleeding. As bleeding persists and the client's level of consciousness declines, breathing will become more shallow and breath sounds will diminish; however, this is a late and unreliable manifestation of bleeding.

A 25-year-old female client seeks care for a possible infection. Her symptoms include burning on urination and frequent, urgent voiding of small amounts of urine. She's placed on trimethoprim-sulfamethoxazole (Bactrim) to treat possible infection. Another medication is prescribed to decrease the pain and frequency. Which is the most likely medication prescribed for the pain? 1. nitrofurantoin (Macrodantin) 2. ibuprofen (Motrin) 3. acetaminophen with codeine 4. phenazopyridine (Pyridium)

4. phenazopyridine (Pyridium) Phenazopyridine may be prescribed in conjunction with an antibiotic for painful bladder infections to promote comfort. Because of its local anesthetic action on the urinary mucosa, phenazopyridine specifically relieves bladder pain. Nitrofurantoin is a urinary antiseptic with no analgesic properties. Although ibuprofen and acetaminophen with codeine are analgesics, they don't exert a direct effect on the urinary mucosa.

During preparation for bowel surgery, a client receives an antibiotic to reduce intestinal bacteria. Antibiotic therapy may interfere with synthesis of which vitamin and may lead to hypoprothrombinemia? 1. vitamin A 2. vitamin D 3. vitamin E 4. vitamin K

4. vitamin K Intestinal bacteria synthesize such nutritional substances as vitamin K, thiamine, riboflavin, vitamin B12, folic acid, biotin, and nicotinic acid. Therefore, antibiotic therapy may interfere with synthesis of these substances, including vitamin K. Intestinal bacteria don't synthesize vitamins A, D, or E.

Renal angiography

An injection of a radiopaque dye through a catheter inserted into the femoral artery to examine the renal blood vessels and renal arterial supply Assess and mark the peripheral pulses. Maintain bed rest and apply a sandbag or other device that will provide pressure to prevent bleeding, if prescribed, at the insertion site for 4 to 8 hours. Instruct the client to maintain a supine position with the leg straight (the head of the bed should not be elevated greater than 20 degrees for 8 hours, (or as prescribed). Assess the temperature, color, movement, and sensation (CMS) of the toes of the involved extremity with each vital sign check. Inspect the catheter insertion site for bleeding or swelling with each vital sign check.

Kock ileostomy (continent ileostomy)

An intraabdominal pouch constructed from the terminal ileum. The pouch is connected to the stoma with a nipple-like valve constructed from a portion of the ileum. The stoma is flush with the skin.

Ureterolithotomy/pyelolithotomy and nephrolithotomy

An open surgical procedure performed if lithotripsy is not effective for removal of a stone in the ureter, renal pelvis, or kidney.

Intravenous pyelography

An x-ray procedure in which an intravenous injection of a radiopaque dye is used to visualize and identify abnormalities in the renal system. Obtain an informed consent. Assess the client for allergies to iodine, seafood, and radiopaque dyes. Withhold food and fluids after midnight on the night before the test. Monitor vital signs. Monitor urinary output.

Pharmacological therapy for urinary incontinence

Anticholinergic agents inhibit bladder contraction and are considered first-line medications for urge incontinence. Several tricyclic antidepressant medications can also decrease bladder contractions as well as increase bladder neck resistance. Pseudoephedrine sulfate (Sudafed), which acts on alpha-adrenergic receptors, causing urinary retention, may be used to treat stress incontinence; it needs to be used with caution in men with prostatic hyperplasia. Hormone therapy (eg, estrogen) taken orally, transdermally, or topically was once the treatment of choice for urinary incontinence in postmenopausal women because it restores the mucosal, vascular, and muscular integrity of the urethra.

Appendicitis

Inflammation of the appendix. When the appendix becomes inflamed or infected, rupture may occur within a matter of hours, leading to peritonitis and sepsis.

Renal biopsy

Insertion of a needle into the kidney to obtain a sample of tissue for examination; usually done percutaneously Preprocedure interventions: Assess vital signs. Assess baseline coagulation studies; notify the physician if abnormal results are noted. Obtain an informed consent. NPO Postprocedure interventions: Monitor vital signs, especially for hypotension and tachycardia, which could indicate bleeding. Provide pressure to the biopsy site for 30 minutes. Monitor the hemoglobin and hematocrit levels for decreases, which could indicate bleeding. Place the client in the supine position and on bed rest for 8 hours as prescribed. Check the biopsy site and under the client for bleeding. Encourage fluid intake of 1500 to 2000 mL as prescribed. Observe the urine for gross and microscopic bleeding. Instruct the client to avoid heavy lifting and strenuous activity for 2 weeks. Instruct the client to notify the physician if either a temperature greater than 100 ° F or hematuria occurs after the first 24 hours postprocedure.

Glomerulonephritis

Term that includes a variety of disorders, most of which are caused by an immunological reaction. Results in proliferative and inflammatory changes within the glomerular structure. Destruction, inflammation, and sclerosis of the glomeruli of both kidneys occur. The inflammation of the glomeruli results from an antigen-antibody reaction produced from an infection or autoimmune process elsewhere in the body. Loss of kidney function occurs.

Creatinine clearance test

The creatinine clearance test evaluates how well the kidneys remove creatinine from the blood. The test includes obtaining a blood sample and timed urine specimens. Blood is drawn when the urine specimen collection is complete. The urine specimen for the creatinine clearance is usually collected for 24 hours, but shorter periods such as 8 or 12 hours could be prescribed.

ileal conduit

Ureters are implanted into a segment of the ileum, with the formation of an abdominal stoma. The urine flows into the conduit and is propelled continuously out through the stoma by peristalsis. The client is required to wear an appliance over the stoma to collect the urine. Complications include obstruction, pyelonephritis, leakage at the anastomosis site, stenosis, hydronephrosis, calculi, skin irritation and ulceration, and stomal defects.

Inflammatory bowel disease

a chronic recurrent inflammation of the intestinal tract that has exacerbations and remissions. Ulcerative colitis Chron's disease

Assessment findings for acute pyelonephritis

a. Fever and chills b. Nausea c. Flank pain on the affected side d. Costovertebral angle tenderness e. Headache f. Dysuria g. Frequency and urgency h. Cloudy, bloody, or foul-smelling urine i. Increased white blood cells in the urine

Assessment finding for chronic pyelonephritis

a. Frequently diagnosed incidentally when a client is being evaluated for hypertension b. Poor urine-concentrating ability c. Pyuria d. Azotemia e. Proteinuria

PostOp colostomy

a. If a pouch system is not in place, apply a petroleum jelly gauze over the stoma to keep it moist, covered with a dry sterile dressing; place a pouch system on the stoma as soon as possible. b. Monitor the pouch system for proper fit and signs of leakage; empty the pouch when one-third full. c. Monitor the stoma for size, unusual bleeding, color changes, or necrotic tissue. d. Note that the normal stoma color is red or pink, indicating high vascularity. e. Note that a pale pink stoma indicates low hemoglobin and hematocrit levels. f. Assess the functioning of the colostomy. g. Expect that stool will be liquid postoperatively but will become more solid, depending on the area of the colostomy. h. Expect liquid stool from an ascending colon colostomy, loose to semiformed stool from a transverse colon colostomy, or close to normal stool from a descending colon colostomy. i. Fecal matter should not be allowed to remain on the skin. j. Administer analgesics and antibiotics as prescribed. k. Irrigate perineal wound if present and if prescribed, and monitor for signs of infection; provide comfort measures for perineal itching and pain. l. Instruct the client to avoid foods that cause excessive gas formation and odor. m. Instruct the client in stoma care and irrigations as prescribed. n. Instruct the client on how to resume normal activities, including work, travel, and sexual intercourse, as prescribed; provide psychosocial support.

Causes of mechanical intestinal obstruction

adhesions following surgery (50%) hernia (15%) cancer (15%) volvulus diverticular disease

Peritonitis

an inflammatory process in the peritoneum that results in extracellular fluid shifts

Interventions for peritonitis

antibiotics NG tube IV fluids analgesics surgery

Interventions for Chron's disease

antimicrobials corticosteroids immunosuppressive agents flagyl B12 injections high calorie, high protein, nitrogen, low fat, low residue diet - no dairy If for surgery then NPO and TPN surgery only done if not responsive to other therapy - high incidence of recurrence - not a cure

irreducible hernia

cannot be replaced into cavity

Causes of constipation

chronic disorders - IBS, diverticular drug induced - antacids, antidepressants, anticholingergics, barium sulfate, iron endocrine - hypothyroidism, diabetes scleroderma neurogenic - megacolon, MS, parkinson's

Causes of non-mechanical intestinal obstruction

neuromuscular vascular disorders (emboli to mesenteric artery) post abdominal surgery inflammatory response (pancreatitis, appendicitis) electrolyte imbalance spinal fracture

Patient teaching after prostate surgery

no intercourse or heavy lifting x 6 weeks avoid sitting/walking for prolonged periods of time drink at least 1-2 L of fluid daily urinate q 2-3 hrs kegels incontinent briefs for dribbling yearly rectal exams

Colostomy irrigation

obtain colostomy irrigation set (cone tip) expel air from tubing position client near toilet instil 500-1,000 mL luke warm water hold irrigation bag 18 - 24 inches above the stoma (shoulder height) lubricate cone and insert into stoma 5 - 10 min in 35 -45 min out stop irrigation if sever cramping

Complications of diverticulitis

peritonitis abscess & fistula formation bowel obstruction urethral obstruction lower GI bleed

Kock ileostomy (continent ileostomy)

surgical intervention for ulcerative colitis An intra-abdominal pouch that stores the feces and is constructed from the terminal ileum. The pouch is connected to the stoma with a nipple-like valve constructed from a portion of the ileum ; the stoma is flush with the skin. A catheter is used to empty the pouch, and a small dressing or adhesive bandage is worn over the stoma between emptyings.

Ileoanal reservoir

surgical intervention for ulcerative colitis Creation of an ileoanal reservoir is a two-stage procedure that involves the excision of the rectal mucosa , an abdominal colectomy, construction of a reservoir to the anal canal, and a temporary loop ileostomy. The ileostomy is closed in 3 to 4 months after the capacity of the reservoir is increased and has had time to heal.

Total proctocolectomy with permanent ileostomy

surgical intervention for ulcerative colitis The procedure is curative and involves the removal of the entire colon. The end of the terminal ileum forms the stoma, which is located in the right lower quadrant.

Urge incontinence

the involuntary loss of urine associated with a strong urge to void that cannot be suppressed. The patient is aware of the need to void but is unable to reach a toilet in time

Stress incontinence

the involuntary loss of urine through an intact urethra as a result of sneezing, coughing, or changing position

Nursing care of nephrostomy tube

urine output is measured separately from each tube q 1-2 hrs ensure tube does not get kinked, clamped, layed on etc. do not irrigate without order - 5-10cc sterile saline - strict aseptic technique daily weights BUN/creatinine


Related study sets

Human Sexuality - Chapter 1 People

View Set

Chapter 3: Using the Internet: Making the Most of the Web's Resources

View Set

SharePoint 2013 Course 20331 Chapter 2 - Designing an Information Architecture

View Set